ChaseDream
搜索
12下一页
返回列表 发新帖
查看: 15881|回复: 10
打印 上一主题 下一主题

备考日记

[复制链接]
跳转到指定楼层
楼主
发表于 2013-8-13 23:15:31 | 只看该作者 回帖奖励 |倒序浏览 |阅读模式
这次考试复习的时候完全乱了pace,东一头西一头整天自己吓唬自己。虽然SC和CR都觉得有突破,但上了考场还是紧张到不行,说明没有复习到位。
老爸电话里说:艺高人胆大,实力上去了自然不紧张。深以为然啊。每天记录一下复习心得,避免再惶惶中不知怎样推进复习进度。

上次用到的材料:
SC: OG, PREP 08, 曼哈顿
CR: OG, PREP 08, 曼哈顿,狒狒一套
RC: OG, 一点点PREP, JJ, 没了...(懒死你..)
数学: Kaplan, JJ
GWD 7套模拟
GMAT单词
杨鹏长难句

看我选材料真是保守啊...最后Verbal的成绩和上次竟然一样...话说上次可是只做了OG就上了考场,真是伤感。成绩出来只觉得两点,一、阅读是硬伤,甚至看见长的改错和逻辑都头疼,二、最后一两个礼拜沉浸在太太乐里面,基础的东西都忘了。其他的原因,暂时还没找到...哭(没找到失败原因怎么进步啊T.T)。

8月13日(二)
单词:以前用过拓词这个软件,觉得不错(虽然有些中文释义不要对它太认真),找出来再背。由于已经背过两三遍,进度很快,不到一个礼拜已经背完1/3,主要作查漏补缺之用。
RC:先加入阅读小分队。由于第23期已经开始了几天,所以每天看两个系列。
SC:开始重新看PREP 08。曼哈顿之前总结过一稿,三个礼拜没看再找出来发现完全没逻辑,重新按照主谓一致、动词的时态和语气、修饰词再整理一遍。明天会总结代词、平行,有时间继续总结比较。


心得:(今天主要集中于语法方面)
1.动名词和不定式:
两者都可以作主语和宾语
但只有动名词可以跟在介词短语后: We are talking about writing in English.
动名词常用于动作 real, concrete,或 completed (stop smoking停止吸烟)
不定式常用于动作 unreal, abstract,或 future (stop to somke我停下手中的事,开始吸烟)

2.in order to 和 to的比较:
in order to 不能作主语,只能作状语,所以不会修饰前面名词

3.to do 和 for doing:
for doing是指某事物的具体用途,而to do则有目的的意思。for doing 有持久性的趋势,而to do往往是一次性行为

4.一道prep上错了两次的题
A new hair-growth drug is being sold for three times the price, per milligram, as the drug's maker charges for another product with the same active ingredient.
A) as
B) than
C) that
D) of what
E) at which
-----------------------------
选C. times作比较时的用法:X times + as +adj./adv.+as; X times +the+n.; X times more than
A选项,as不可以单独出现


5.在论坛上看到几个查漏的帖子以及牛牛们的回复
OG13-107. While it costs about the same to run nuclear plants as other types of power plants, it is the fixed costs that stem from building nuclear plants that makes it more expensive for them to generate electricity.
(A) While it costs about the same to run nuclear plants as other types of power plants, it is the fixed costs that stem from building nuclear plants that makes it more expensive for them to generate electricity.

(B) While the cost of running nuclear plants is about the same as for other types of power plants, the fixed costs that stem from building nuclear plants make the electricity they generate more expensive.
(C) Even though it costs about the same to run nuclear plants as for other types of power plants, it is the fixed costs that stem from building nuclear plants that makes the electricity they generate more expensive.
(D) It costs about the same to run nuclear plants as for other types of power plants, whereas the electricity they generate is more expensive, stemming from the fixed costs of building nuclear plants.
(E) The cost of running nuclear plants is aboutthe same as other types of power plants, but the electricity they generate ismade more expensive because of the fixed costs stemming from building nuclear plants.
---------------------------------------
这里是比较Thecost of running nuclear plant the cost of running other types of power plants
简化为A of B is the same as C, 尽管意思上没有问题, 但在语法上,C是和主语A比较,还是和介词短语限定的A of B比较,不明确。 如果选项E是about the same as OF other types of power plants,我想你一定不会有疑问。但这个题的可恨之处在与他用了一个意思相近别的介词短语A  for B 来代替 A of B.
选B
stem from起源于

PREP 44. The Environmental Protection Agency frequently puts mandatory controls on toxic substances that present as little risk as one in a million chances to cause cancer.
A. as little risk as one in a million chances to cause
B. as little risk as one chance in a million of causing
C. as little risk as one chance in a million that it will cause
D. a risk as little as one chance in a million for causing
E. a risk as little as one chance in a million for it to cause
-----------------------------------------------------------------------------
选B。chance of (doing)sth, chance to do sth; risk of doing,risk of sth,risk doing--》不是to do!,也不是for doing
另外看到for doing 除非是很肯定表示用途,多半是错的。
-----------Ron的原文
whoa, you guys are missing the main point here:the word whose idiomatic usage is being tested is risk, not chance.

this is a bit hard to see in this particularsentence, so here's an analogy (which i'm making up on the spot - not part ofan official question):
as small a collection as three pirated albums has occasionally drawn the attention of the recording industry.
in this case, 'collection', not 'albums', is thesubject of 'has drawn' (which can be inferred from the fact that 'has' issingular).
this is the case because this sentence is equivalent to the following rearranged version:
a collection as small as three pirated albums has occasionally drawn the attention of the recording industry.
the same reasoning applies here; you're looking for idiomatic usage that agrees with 'risk', not 'chance'.

the last poster is correct in one sense, which isthat there are correct idiomatic usages of 'chance to'.
HOWEVER,
the last poster is incorrect in this particular scenario, because 'chance to' is NOT used when 'chance' refers to amathematical probability (as it does in this context). in the case of mathematical probabilities, you can only use 'chance of'.
for instance, you can't say this treatment has a70% chance to cure the disease; you have to say chance of curing.

OG13-35
By 1940, the pilot Jacqueline Cochran held seventeen official national and international speed records, and she earned them at a time when aviation was still so
new for many of the planes she flew to be of dangerously experimental design.
(A) and she earned them at a time when aviation was still so new for many of the planes she flew to be
(B) earning them at a time that aviation was still so new for many of the planes she flew to be

(C) earning these at a time where aviation was still so new that many of the planes she flew were
(D) earned at a time in which aviation was still so new such that many of the planes she flew were
(E) earned at a time when aviation was still so new that many of the planes she flew were
-----------------------------------------------------------
OGB项的解释是:Here, the word earning takes the pilot herself, not the records, as its subject. However, earning is close to the records, not to Jacqueline Cochran, making this sentence hard to process. In addition, the causal relationship between aviation being new and planes being of experimental design is more effectively communicated by the structure so new that many X were Y, not by the structure so new for many Y to be X.
答案是E
"按照Ron的条件,earning them at a time完全满足。earning可以视为和held同时发生,且apply to  the subject。也就是对的,为什么OG却说是不对的,有可能修饰records."
从语义上讲,后面那一串,讲的是internationalspeed records怎么earned的,所以用 comma + v-ed 作为noun modifier修饰international speed records.

6.最后贴上上次考试前才发现的悠悠阅读法,这次要用心体会一下:
大到任何书籍,任何文章,小到任何句子,任何词语,要想读透,一定要知道弄清楚两点:

author’s intent [作者的意图],
writer’s craft [作者的手法]。

为什么?因为弄清楚这两点之后,你就变成作者本人了。
意图是你动笔的目的,手法是你为了传达你动笔的目的所用的词、句、例子、理论、逻辑、引用、等等。


你会发现GMAT阅读的考题里,问到的问题大多数都是这样的类型:
作者为什么写这篇文章?
作者为什么举这个例子?
作者引用这段话的目的是?
你可以推测作者认为xyz?
文章里高亮的作者的这句话在整篇文章里的作用是?
……
由此可见弄懂作者的意图和手法对做题是多么重要。

-------------------------------

随便打两个简单的比方。

如果一段话这么写:
观点1……例子……观点2。
作者的意图是什么?
肯定是支持观点1,观点2和观点1一定是同一战线上的,只不过是对观点1所作的延伸。
我是怎么知道作者的这个意图的呢?
因为作者用了例子这个手法。在观点1和观点2之间,举个例子作为观点1的支撑,并作为把观点1和观点2联系起来的桥梁。
(类似写法:OG13 第380页第一段话)

现在,如果一段话这么写:
观点1……观点1的逻辑……转折(观点2)……例子……观点3。
作者的意图是什么?
肯定是指出观点1逻辑上的问题,并且支持观点2。
我是怎么知道作者的这个意图的?
因为作者的手法!因为作者一把观点1的逻辑写出来,马上就做了个转折,说明观点1的问题一定出在它的逻辑上,这也就是观点2的内容。接着作者举例,这个例子肯定是用来证明观点1逻辑上的错误(证明观点2),进而支持观点3的。
(类似写法:OG13 第386页)

想想如果是你自己写文章,是不是也得这样写?!
这样你还能顺藤摸瓜,想到接下来文章作者要继续写的话,是不是应该拓展观点3的某些方面?这有助于对下段的理解。


GMAT很可能会这么考你:
观点1[考点:作者对观点1持什么态度]……观点1的逻辑[考点:我们可以推测这句话说明什么?]……转折(观点2)[考点:这句话在文中的作用是?]……例子[考点:作者举这个例子的作用是?]……观点3[考点:我们可以推测作者认为……?]
像这样的考题,只要你清楚作者的意图和手法,就完全不在话下。


8月14日(三)
阅读小分队,单词照旧。小分队今天做了3组,终于赶上进度,但读完的东西好像不应该做完了就算啊,是否应该再回顾一下,起码应该再提炼一下结构?

其中一个楼主jay871750293 给我的回复:
我认为仅仅是计时+写总结其实效果并不会太好(我现在就是这样的...),我认为这样充其量算是每天给自己一个英语的环境,可以防止阅读能力下滑,但如果想要提高的话,还要做得更多,比如记生词、反复看长难句,熟练意群读法等等;至于读速的话,是根据文章的难度不同而不断发生变化的,当然目标是150wpm(保证一定的理解能力),我个人认为100wpm应该算比较慢了吧(自己感觉的,不一定准确),不过我认为不用纠结现在的速度,只要以150wpm为目标就好了哈~
总结是否到位的话,可以看看其他大牛的总结,比如版主yingjie姐姐的总结,很赞的哟~
最后,至于每句话是否需要读懂,我是这样想的:如果可以在保证150wpm同时,读懂每句话,那么就是完美了;但如果达不到的话,也不一定非要读懂每句话,毕竟每一段话都有关键与非关键句,对于文章段落或文章整体意思理解没有什么影响的非关键句子略读甚至跳读也是可以的,比如科技文里经常出现的对教授的介绍、有些文章中的类比或是类似的举例、还有就是破折号或者是括号里的内容,有些重点与非重点读得多了也就比较容易判断了~

语法PREP08约50道题。
又看见一个帖子说应该认准一个语法点,然后刷OG,也就是说仅用这个语法点排除选项,加深对该语法点的理解。正在考虑是否用这个方法再刷OG。


心得:
1. GWD阅读方法摘抄
首先读文章
(1)读段首
(2)读转折:如转折太多,则只关心最后转折
(3)全文最后
(4)长文章,一般为四段,关注段首+全文最后
观点类考题
(1)找废话。如果有且只有一个选项内容可以不依赖原文存在,则该选项为正确,因为“虚的”“泛的”即为正确
(2)当选项相似或接近时,原文失去所有意义,只有当选项不相似时,原文才是有用的
(3)选项出现比较(more than),则按错误选项对待(仅适用观点类考题,正确率85%以上)
(4)互斥选项,有一个是对的
细节类考题
(1)根据问题问什么去找,如果问题没得找,再用选项找
(2)不能找虚的
(3)找名词为主,其他词会变
(4)不看意思,只看词的长相,找到后,整段找完就结束
(5)找到后,记忆大于理解,忘掉别处,记忆信息比理解重要


2. attempt
作及物动词时:attempt to do sth.
                      attempt sth.
作名词时:an attempt at (doing) sth.
                an attempt to do sth.—不如attempt to do sth.简洁有效

3.PREP08总结
#1-222: Although people in France consume fatty foods at a rate comparable to the US, their death rates from heart disease are far lower in France.
(A) people in France consume fatty foods at a rate comparable to the US, their
(B) people in France and the US consume fatty foods at about the same rate, the
(C) fatty foods are consumed by people in France at a comparable rate to the US's, their
(D) the rate of fatty foods consumed in France and the US is about the same, the
(E) the rate of people consuming fatty foods is about the same in France and the US, the
------------------------------------------------
正确B。D选项中 the rate of fatty foods 错,不是foods的rate,而应该是consuming foods的rate。consumed in France and the US错误,应该在the US前面加in,同时主语改为rates。A选项中their为完全指代,即people in France,带入句中语义重复

#1-223
After several years of rapid growth, the health care company became one of the largest health care providers in the metropolitan area, while it then proved unable to handle the increase in business, falling months behind in its payment to doctors and hospitals.
A.while it then proved unable to handle the increase in business, falling months behind in its payment to
B.while it then proved unable to handle the increase in business and fell months behind in its payment to
C.but then it proved unable to handle the increase in business, falling months behind in its paying
D.but then proving unable to handle the increase in business, falling months behind in paying
E.but then proved unable to handle the increase in business, falling months behindin paying
---------------------------------------
选E。C中的in its paying,使得paying从动词变成了名词,不简洁,且paying这个动作就是the health care company执行的,所以不用加上its. prep.+one's+doing往往不简洁

#2-164
Mixed with an equal part of water, ethylene glycol, a compound commonly used as an automotive antifreeze, is effective at temperatures as low as –30degrees Fahrenheit.
A.temperatures as low
B.temperatures so low
C.as low temperatures
D.as few
E.as little
---------------------------------------------------------
选A。as adj. (a) noun. as = (a) noun. as adj. as,其中第一个as是副词。C的问题是at后面缺少宾语

#2-186
The greatest road system built in the Americas prior to the arrival of Christopher Columbus was the Incan highway, which, over 2,500 miles long and extending from northern Ecuador through Peruto southern Chile.
A.Columbus was the Incan highway, which, over 2,500 miles long and extending
B.Columbus was the Incan highway, over 2,500 miles in length, and extended
C.Columbus, the Incan highway, which was over 2,500 miles in length and extended
D.Columbus, the Incan highway, being over 2,500 miles in length, was extended
E. Columbus, the Incan highway was over 2,500 miles long, extending
-----------------------------------------------------------------
正确E.关于D,being使用wordy; in length 不如 long 简洁;extended 被动形式错,句意变成了 highway 被延长了,应该是“一路延伸到某个长度”
关于being: (如果表达人或者物的身份和性格,应该尽量避免使用being,因为常常多余)
Ron的总结:
the short answer to this question: basically, "being" is ok when there are no acceptable alternative formulations that DON'T use it.
from what we've seen, "being" can be ok as long as at least one of the following 2 conditions obtains:
(1) it is part of a PASSIVE-VOICE construction (note that such constructions require a form of "to be", so, if the passive voice is in the -ing form, that form will manifest as "being");
or
(2) it is used as a GERUND (i.e., the action of "being something" is treated as a NOUN in the sentence).
most, but not all, of these gerunds will be the gerund form of a construction in the passive voice.

examples: (the first one i made up; the other three are correct answers to official problems)
Being followed by paparazzi 24 hours a day has caused many celebrities to become extremely hostile to strangers. (source: i made this up on the spot)  
(note that this is both --> it's a passive-voice gerund!)
During the 1950s, as part of their therapy, young polio victims learning to live with their disabilities were helped to practice falling, so that they could learn to fall without being hurt.  (source: gmat prep)
(again, passive voice gerund)
According to one expert, the cause of genetic irregularities in many breeds of dog is not so much that dogs are being bred for looks or to meet other narrow criteria as that the breeds have relatively few founding members. (source: gmat prep)
(passive; not gerund)
Being heavily committed to a course of action, especially one that has worked well in the past, is likely to make an executive miss signs of incipient trouble or misinterpret them when they do appear. (source: gmat prep; also in the OG verbal supplement)
(gerund; not passive)

再补充prep上一题:#1-235
A mixture of poems and short fiction, Jean Toomer's Cane has been called one of the three best novels ever written by Black Americans—the others being Richard Wright, author of Native Son, and Ralph Ellison, author of Invisible Man
A.Black Americans—the others being Richard Wright, author of Native Son, and Ralph Ellison, author of Invisible Man
B.Black Americans—including Native Son by Richard Wright and Invisible Man byRalph Ellison
C.a Black American—including Richard Wright, author of Native Son, and Ralph Ellison, author of Invisible Man
D.a Black American—the others being Richard Wright, author of Native Son, and Ralph Ellison, author of Invisible Man
E.a Black American—the others being Richard Wright's Native Son and Ralph Ellison's Invisible Man
-----------------------------------------------------
正确E,the others being使用正确。关于用Black Americans还是用a Black American,GMAT似乎倾向于后者,更加清晰指出一本书是一个人写

4. such A as B 的结构,谓语单复数情况和A一致,因为B只是举例的内容

5. 限定性名词修饰:无逗号隔开,用that — 如果介词提前需转用which,不用加逗号
非限定性名词修饰:有逗号隔开,用which
限定性与非限定性的区别:有该修饰时,修饰对象的外延是否有变化,如果变小了就是限定性,如果没变就是非限定性。
                                      所以要根据范围是否需要变化,来决定用限定或非限定。

6. media 是 medium 的复数形式

7. news 是不可数名词,谓语动词用单数

8. will+动词原形,除了表示将来时,还可以表示常常发生的事情

9. 查漏补缺题:
Some anthropologists regard the early hominids' manner of walking as being less efficient than in modern human beings.
(A) as being less efficient than in
(B) as less efficient than it is in
(C) as less efficient than that of
(D) to be less efficient than that of
(E) to have been less efficient than it is in
---------------------------------------------------------
选C。as 是 regard as习语的一部分,和than没有关系。D就错在regard to be错误。
参考:
http://www.manhattangmat.com/forums/some-anthropologists-regard-the-early-hominids-manner-of-wa-t2266.html

收藏收藏2 收藏收藏2
沙发
 楼主| 发表于 2013-8-15 20:20:45 | 只看该作者
本来想把一个礼拜的内容放在一个帖子里面,发现太难找了,有时还不小心把前面的内容给删了,所以单发了...
8月15日(四)
单词:已经背超过一半了,yeah,鼓励一下自己。但是仍有一些形似的单词每次都会搞混,有时间要总结下。准备了单词卡背诵阅读小分队中遇到的生词。
阅读小分队:今天继续跟帖,并开始总结以前做过的小分队两篇,和牛人的总结对照。
语法:继续Prep08,50道左右,过完第一遍,开始第二遍,很多题还没能细心体会,会在下面摘抄几道。


心得:
1. Prep08上的难题
#2-193:As the honeybee’s stinger is heavily barbed, staying where it is inserted, this results in the act of stinging causing the bee to sustain a fatal injury.
A.As the honeybee’s stinger is heavily barbed, staying where it is inserted, this results in the act of stinging causing
B.As the heavily barbed stinger of the honeybee stays where it is inserted, with the result that the act of stinging causes
C.The honeybee’s stinger, heavily barbed and staying where it is inserted,results in the fact that the act of stinging causes
D.The heavily barbed stinger of the honeybee stays where it is inserted, and results in the act of stinging causing
E.The honeybee’s stinger is heavily barbed and stays where it is inserted, withthe result that the act of stinging causes
----------------------------------------------
选E。原文应指蜜蜂的刺倒钩,并留在扎入的地方,导致了后面的结果,但C选项是蜜蜂刺本身导致了后面的事,逻辑不合理;staying where it is inserted作非限定性修饰,暗示了stinger无时无刻都是staying的,不合理;result in the fact that the act of,wordy。D选项的the heavily barbed stinger of the honeybee results in逻辑错误同C选项;result应该用伴随状语来表示结果。(这道题的区别点需要细腻地体会...)

#2-204:ANew York City ordinance of 1897 regulated the use of bicycles, mandated amaximum speed of eight miles an hour, required of cyclists to keep feet onpedals and hands on handlebars at all times, and it granted pedestriansright-of-way.
A.regulated the use of bicycles, mandated a maximum speed of eight miles an hour,required of cyclists to keep feet on pedals and hands on handlebars at alltimes, and it granted
B.regulated the use of bicycles, mandated a maximum speed of eight miles an hour,required cyclists to keep feet on pedals and hands on handlebars at all times,granting
C.regulating the use of bicycles mandated a maximum speed of eight miles an hour,required cyclists that they keep feet on pedals and hands on handlebars at alltimes, and it granted
D.regulating the use of bicycles, mandating a maximum speed of eight miles anhour, requiring of cyclists that they keep feet on pedals and hands onhandlebars at all times, and granted
E.regulating the use of bicycles mandated a maximum speed of eight miles an hour,required cyclists to keep feet on pedals and hands on handlebars at all times,and granted
---------------------------------------------------
A NYC ordiance regulating sth. mandated..., required..., and granted...
选E。其中B选项regulate与mandate,require并列逻辑意思不合理;三个动词并列,最后一个动词前应该加and。C选项中require that从句用动词原形表示虚拟语气。(也是需要细腻体会的题,谁和谁并列?)

关于require的用法:
(1)require sb./sth. to do sth. (不定式可以用被动形式) 错误的表达方式:require sb. doing
(2)require (of sb.) that + 动词原形    需要某人做某事
        大全475:Legislation in the Canadian province of Ontario requires of both public and private employers that pay be the same for jobs historically held by women as for jobs requiring comparable skill that are usually held by man.
(3)N is required (of sb./sth.) to do st. 某物对某人做某事来说是必须的
      或 N is required of sb./sth. to do (其中required是过去分词)
        大全166:Because of the enormous research and development expenditures开支,支出 required of firms to survive in the electronics industry, an industry marked by rapid innovation and volatile demand, such firms tend to be very large.
        Windmaple的例句:This leaves open an "in-between" position: The efforts required of someone to prevent the consequences of his non-negligent act could be greater than what would be required of any bystander旁观者,看热闹的人, but not as great as would be required to avoid an act than is known to cause the harmful consequences.
(4)require doing (require主动表被动含义):如 A require making known to B A需要被B知道
*GMAT中出现过的错误表达:
   require of sb. to do(require of sb. 后面应该加 that,只有require sb.和sth. is required of sb.后面才是加 to do 的)
   require sb. that(require 若和 that 搭配,则跟sb.之间要有of)
   require sb. doing..错误

关于命令性的虚拟语气:主语+命令性词汇+that+从句主语+动词原形+...
(1)常见的命令性词汇(名词形式同样应用虚拟):demand, dictate命令,口述,insist, mandate, propose, recommend, request, stipulate规定,保证,suggest
(2)有些词汇只能用不定式:advise, allow, forbid, persuade, want
(3)有些既可以跟that也可以跟不定式:ask, beg, intend, order, prefer, urge, require
(4)形容词性的命令词汇可跟that也可以跟不定式:essential, advisable, crucial, desirable, fitting, imperative, important, mandatory, necessary, preferable, urgent, vital至关重要的
(5)例外:prohibit: prohibit sth.; prohibit from doing; prohibit sb. from doing. 但是不能接不定式
注:如果动词不表示命令语气则例外:Her presents suggests that she is happy.


#1-174:Often major economic shifts are so gradual as to be indistinguishable at first from ordinary fluctuations in the financial markets.
A.so gradual as to be indistinguishable
B.so gradual so that they can be indistinguishable
C.so gradual that they are unable to be distinguished
D.gradual enough not to be distinguishable
E.gradual enough so that one cannot distinguish them
----------------------------------------------------------
正确选A。so+adj.+as to be/do一般用于单一主语,但是不能相隔太远。C选项中,able/unable to be done结构错误

#1-183:Despite recent increases in sales and cash flow that have propelled automobile companies' common stocks to new highs, several industry analysts expect automakers, in order to conserve cash, to set dividends more conservatively than they were.
A.to set dividends more conservatively than they were
B.to set dividends more conservatively than they have been
C.to be more conservative than they have been in setting dividends
D.that they will be more conservative than they were in setting dividends
E.that they will be more conservative than they have been to set dividends
--------------------------------------------------------
正确选C,be+adj.+in doing something正确。D,E选项中不能使用expect sb. that的搭配,只能用expect that或expect sb. to do。
关于expect的用法:
(1)意思为“预料”时:sth. be expected to do/be 指望,期望
                          it is expected that 预期(和上一个意思一样,但没有上一个简洁)
                          expect to do sth.
                          expect sb./sth. to do sth.
                          expect that
                          expect sth.
(2)意思为“想要”时: expect sth. from sb.
                                expect sb. to do sth.
(3)错误用法:sth. be expected for sth. to do(也就是说不能加for sth.)
                        sth. be expected that 错
                        it is expected for sb. to do 累赘,不如直接用sth. be expected to do/be
                        expect sb. that 错


2.阅读小分队总结
第四期系列23第一篇科技:我的笔记http://forum.chasedream.com/forum.php?mod=viewthread&tid=868402&page=13#pid18473311
Time 1: 2'08"
Scientists found out why sunburn hurts. Some molecule caused this pain involved in many other pain process.
Time 2: 3'30"
Scientists used a experiment to prove UVB exposure activates TRPV4 in mice. Blocking TRPV4 will prevent and treat sunburn.
脉络:a phenomenon, sunburn hurts - the benefits of the hurts - why it hurts, an experiment in mice - can it apply to human? - need further research on protection against UVB exposure
NN的总结:Researchers  discovered that sunburn hurt is caused by a molecule,  TRPV4, which is abundant in the skin's epidermis after longtime sun exposure.
                 Blocking the TRPV4 may provide certain protections against UVB exposure.
epidermis上皮,表皮
mutant突变体,突变的(科技文中出现多次)
dissect进行解剖,仔细分析
具体的原因:Further molecular analysis uncovered the entire sequence of events in this pathway, with each event affecting the next: UVB exposure activates TRPV4, which causes the influx of calcium ions, which brings in another molecule called endothelin, which triggers TRPV4 to send more calcium into the cells. Endothelin is known to cause pain in humans and also evokes itching, which could explain the urge sunburned patients feel to scratch their skin.


Time 3: 2'36"
The sunshine can help people reduce blood pressure, and the benefits far outweigh the risk of getting skin cancer.
Through a experiment scientists proved that sunshine not the heat can reduce the blood pressure.
But scientists still need to look at the relative risk of heart disease and skin cancer.
Good enough.
NN的总结:It reveals that the benefit to heart health(less blood pressure) of sunlight may outweigh the risk of skin cancer.


Time 4: 1'06"
Documentaries are more helpful on memory or attention improvment than are mental games.
Time 5: 2'14"
Scientists did not find a huge impact on mental training games, but they think maybe because of the reasons below:
The documantaries are focus on language but only 1/5 of the games is focus on it.
The team hasn't do enough training. But scientist still need to find out what is 'enough'.
补充:A discussion on whether mental games are effective at boosting mental capacity - maybe not as useful as documentaries - but..maybe somethings affect the result (作者最后的结论:可能还是有用,但还没发现怎么更有用?原文:Despite the results of the study, Borness says she herself is still a user of brain-training games. "I think they're fun. I'm one of those people who can't do nothing, so I get on my phone and play games.")
NN的总结:A test shows that the brain-training games may slightly improve the language skill, but have no significant role in boosting multiple measures of intelligence or cognitive function, which the brain games were originally designed for.(好像...)


Time 6: 5'40"
Whether an impact extinct P period? Some scientists believed that even the extinction caused by an impact, there are still have some different reasons with C period extinction. But some scientists believed the crafter found as a evidence is just a coincidence.
补充:at least five big extinction happened - however, whether had the same reason with dinosaurs at the end of C period? - the biggest one, at the end of P period, was triggered by an impact too - but a scholar said the details were different - maybe because of huge volcanic eruptions? and for the impact theory still miss a crater - the scholar found one, which althoug much smaller than the one impact at the end of C period, still triggered big earthquake, released gas and oil, and generated a huge burp打嗝 of methane into the atmosphere - still need further research
(最后一段总结得很重要Determining whether this was really what happened will take a lot more digging, of course. Even now, there are those who think the formation of the Chicxulub crater was a coincidence, and that what did for the dinosaurs was actually the volcanoes, so Dr Tohver will have to work hard to convince the sceptics. If he does, though, he will have proved himself a great geological detective, for he will have been responsible for solving one of the biggest puzzles in palaeontology.)
小心得:however, but这些很重要;时间之类的也很重要,如C period, P period,表示不同的历史事件;最后一段很重要
upshot 结局,要点
culprit 犯人,被控犯罪的人(出现过,往往用于原因推断的时候)
NN的总结:
MI: The mass extinction may related to a space rock.
Structure:
=>Why dinosaurs were wiped out?
=>Ideas raised by Dr. Tohver that the Permian extinction was related to meteorite.
=>The finding of A. crater, which might be the evidence of the culprit of the mass extinction.
=>The further explanation for the mass extinction and its criticism:
oil shale->biggest fracking operation->burp of methane->too hot to live for the dinosaurs in the Permian period.


系列23第二篇科技,我的笔记:http://forum.chasedream.com/forum.php?mod=viewthread&tid=868485&page=10#pid18474704
Time 1: 1'29"
A new product mixed coffee and alcohol was invented. Researchers thought it is good enough for market.
concoction混合,混合物
ferment发酵,动乱
coffee grounds 咖啡渣,磨碎的咖啡
pungent辛辣的,刺激的,苦痛的

NN的总结:A way is found to turn the coffee grounds into alcoholic beverage, which has no caffeine left after the brewing process.

Time 2: 2'05"
A experiment shows dogs will lift their left eyebrow if they see their owners or are happy. If they are learning, they probably twitch their right ears, because the process is controlled by the left brain.
一个转折的引导:Some of the results are puzzling, ...why the positive emotion did not show on the right face? - a mixed emotion, happy to see its owner, sad because cannot touch him/her
(推测原因的原文,注意while的用法:It may be due to mixed emotions—something dogs were not known to experience. While seeing their owners made the animals happy, they were blocked by the partition from reaching out for that all important touch—which made them sad. 好Q~)
NN的总结
:You can tell the love from your dog by observing the lift of its left eyebrows.

Time 3: 1'20"
Food with high fiber can prevent cardiovasular disease, especially from women.
The reason of difference between gender hasn't been found out, and scientists concluded that it maybe woman consume more high fiber food and that they will pay more attention on gender when research the diet issues.
(原文:虽然性别的原因还不清楚,但作者继续用对比提出假设的原因:However, a probable explanation is that women consume fibre from healthier food sources than men do. Women ate a lot of fibre in the form of fruit and vegetables, whereas the most important source of fibre for men was bread.)
Time 4: 1'56"
The researcher hasn't found any solid links of other nurtrients, such as suger, with cardiovasular disease.
For the effect of people controlling their diet, the researcher said they knew too little about the effect of dramatically change food structure.
NN的总结:The intake of fiber may lower the risk of cardiovascular disease, esp. for women.
                The link, however, between the cardiovascular disease and fat/sugar is still unclear. The extreme diet recommended by National Food Administration may benefit you in the short run, but not necessarily in the long run.


Time 5: 1'42"
Not sure about the topic of this passiage..
原文链接如下,有空再读读...:http://news.sciencemag.org/2013/08/scienceshot-graveyard-iceballs-past
A phenomenon - some objects in the asterroid belt spew water or dust recently
Reason - old view: maybe they've collided with something.
          new view: most is remnants of comets without surface ice.
开始没读懂的句子:Now, scientists propose that most if not all of these objects may be shedding dust or water vapor because they’re the barely active remnants剩余,剩余的 of comets that are now largely bereft丧失的 of surface ice.)
NN的总结:The shedding vapor or dust of the objects in the asteroid belt may come from the the active remnants of comets.

Time 6: 6'55"
Some people died when they participated the extreme sports.
Tips from professionals:
Do not overestimate your ability and practice.
Dress smart.
Notice the early signal from your body, such as cramping.
grueling折磨人的,令人筋疲力竭的

最后附上iamyingjie 介绍阅读方法的帖子,没有诀窍,只有坚持,永远不要觉得自己足够努力...再激励自己一下!有时间去膜拜:
没有诀窍。就是每天认真阅读,长期坚持。
我给你看几篇我发的作业帖子,以及我觉得我做的还算是比较认真的作业吧。你就可以明白我在说什么了。
1. 4月19日_17-15: 亮点在于沙发(2#)部分的单词,以及板凳(3#)部分的精读作业。
http://forum.chasedream.com/thread-837794-1-1.html
2. 3月30日_16-30: 重点在于沙发(2#)的单词表。
http://forum.chasedream.com/thread-827938-1-1.html
3. 6月20日_20-16_Mindset: 神猴找的文章本身很有启发意义。我的作业在7#, 你也可以参考一下。
http://forum.chasedream.com/thread-860038-1-1.html
4. 7月4日_21-10: 我的又一次精读尝试。详见8#
http://forum.chasedream.com/thread-863426-1-1.html

3. 语法的查漏补缺:
OG12-19: The Iroquois were primarily planters, but supplementing their cultivation of maize, squash, and beans with fishing and hunting.
(A) but supplementing
(B) and had supplemented
(C) and even though they supplemented
(D) although they supplemented
(E) but with supplementing
---------------------------------------------
选D。A选项中的The participle supplementing would normally be expected to modify the first clause, describing or extending its meaning, but the logic of this sentence demands a contrast, not an extension。同时,即使考虑省略,时态不对,而且前一个were是系动词,后一个were是助动词。

8月16日(五)
时间过得好快,距离上次考试已经一个礼拜,继续加油!
单词继续。
阅读小分队跟帖,并读了iamyingjie建议的精华前两组,有楼主的单词总结和背景介绍,赞。总结一篇。
语法继续Prep08,50-70道左右。上次Prep08就只过了两遍,恐怕体会不够深,这次一个星期已经过了一遍半,每次都有新体会。


心得
1.阅读小分队第九篇科技,我的笔记:http://forum.chasedream.com/forum.php?mod=viewthread&tid=869870&page=5#pid18488196
Time 3: 2'31" 295 words
Nowadays, many people eat Chinese herbal supplements for their health, but authorities found an ingredient known as birthwort in the supplement can be cancerous.
Time 4: 1'47" 274 words
You still can get birthwort from Chinese practitioners or internet. A researcher from Singapore studied the case in Taiwan and found 1500 genes with mutations (means eating birthwort is more dangerous than smoking or exposure to ultraviolet radiation).
Time 5: 1'23"
Birthwort is responsible not only for upper urinary tract cancer but also for liver cancer.
(然后一句没读懂的:Someday, the first clue that a patient has ingested birthwort could come not from the person’s clinical history, but from the herb’s powerful genetic fingerprint, he says. If found in patients living in places not widely known to have birthwort exposure, such a fingerprint could be a sign that public health campaigns are needed.)
NN的总结:A link is found between the kidney damage and the usage of birthwort, a herbal supplement containing aristolochic acid.
         A mutation of gene, called p53, is found to be the cause of cancer though the use of supplements w/ birthwort is linked with urinary tract cancer.
         Being exposed to the birthwort is dangerous and it should call for the awareness of the public.
好句子:Soon after, scientists discovered a link in the rural River Danube region between kidney damage and wheat that had been contaminated with birthwort when seeds from the two plants mixed during the harvest. (two plants - wheat & birthwort)
          But such supplements remain available from traditional Chinese practitioners and over the Internet, notes Bin Tean Teh, a cancer researcher at the National Cancer Centre Singapore and the National University of Singapore (NUS) and the corresponding author on one of the new papers, which appear this week in Science Translational Medicine. (supplement still available - notes Bin)
关于没读懂的这一句楼主的回复:
根据你提到的问题,我在读第一遍的时候也有一些疑惑.
速度的3,4,5是一篇文章.所以我建议最好结合前文来理解.在速度4,有提到虽然有一些研究证明birthwort和upper urinary tract cancer or kidney disease是有联系的,比如一种基因突变P53。但是实际上, 后面的研究发现这种突变时多达1500 genes with mutations---a level higher than has been found for either lung cancer in smokers or skin cancer in patients with heavy exposure to ultraviolet radiation.这说明食用birthwort确实会引起人类基因的改变。
你提到的速度5的这句话:
the first clue that a patient has ingested birthwort could come not from the person’s clinical history, but from the herb’s powerful genetic fingerprint.
说一说我的理解:既然birthwort会对人体造成伤害(可能患上癌症),那么在这个过程中应该会有前面提到的大量基因突变,所以当我们发现一个人被诊断出birthwort特有的基因指纹(速度4中的解释:leave genetic fingerprints on a person’s genome, in the form of characteristic alterations in the letters that make up DNA—sometimes thought of as “spelling mistakes.”)造成的基因突变时,这个线索也就非常能够有助于判断一个人是否has ingested birthwort了。至于这句话说“第一线索不是来自于这个人的病例”,我想既然第一段提到人们把含有birthwort的herb(草药)当作有助于健康的药物使用,那么可能当一个人生病的时候就会服用这种草药吧。
但问题的关键在于服用了这种草药并不能说明这个人生病就是由birthwort引起的,而应当被检测出由birthwort带来的基因突变才能说明这个人生病确实是由含有birthwort的herb引起的!


2.estimate的用法:
    estimate + that 不需要加动词原形
    estimate + N +to be N (adj.)
    estimate + N + at/to be + 数量词
*曼哈顿认为估价的话要用to be
  right: She estimates the cost to be ten dollars.
  wrong: She estimates the cost at ten dollars.

3. tend 的用法:
     tend to do sth - if something tends to happen, it happens often and is likely to happen again
     tend to sb/sth 照顾
    *错误用法:tend doing

4. 累赘的词汇
from ... (down/up) to
rise (higher)
then .. (subsequently)
now .. (currently)
(to someone,) Paris is someone's home
enable someone to (be able to)
continue, and (have already done)
(up) until..
per capita consumption .. (per person)
each year ... (annually)
(consistently) .. doing sth.
also ... (as well)
the more ..., the more, become (increaseingly)
there is (continuous) sth. occurring
someone is doing sth. (continuously) 总之continuous类不要和现在进行时连用
(so) in order to
then (later)
then (subsequently)

5.直接加+宾语+宾补的搭配:
consider, deem, make, call, find, elect + 宾语 + 宾补

6.加as的搭配:
use, cite, function, regard, think/conceive of, view, act + as

8月17日(六)
单词:2400多单词,已经掌握2000+啦,goodgood
阅读:阅读小分队跟帖一组,小分队精华一组,开始63篇第2-4篇。之前做的GWD的模考题中阅读整理的也不好,在考虑要不要整理,还是从Prep08先开始?
语法:过完Prep08第二遍。

心得
1. mumuwa的阅读技巧示范,值得反复看
http://forum.chasedream.com/forum.php?mod=viewthread&tid=106437&highlight=mumuwa

2. 63篇...把我打击到了,怎么觉得做GWD和正式考试都没有这么难?也可能是我分不高没碰上难题的原因。
63篇的总结帖子以及新东方课堂笔记:http://forum.chasedream.com/forum.php?mod=viewthread&tid=520057&highlight=63%E7%AF%87
第二篇9道题只对了2,4,5... 里面好多句子是从杨鹏长难句来的啊...论坛上找到的关于第二篇的分析,有启发,需要再体会两遍。
http://forum.chasedream.com/forum.php?mod=viewthread&tid=575569&highlight=63%E7%AF%87
defy挑战,藐视,对抗
gyroscope陀螺仪,回转仪
windmill风车,做风车般旋转
treadmill跑步机,单调的工作
bellows风箱,皮老虎
perpatuate永恒化
据说的主题句:There has been no boasting about our social workers - they are merely signs of the system's failure, of opportunity denied or not taken, of things to be eliminated. We have no pride in our growing interdependence, in the fact that our system can serve others, that we are able to help those in need; empty boasts from the past make us ashamed of our present achievements, make us try to forget or deny them, move away from them. There is no honor but in the Wonderland race we must all run, all trying to win, none winning in the end (for there is no end).


3. 63篇第三篇自己的分析
No very satisfactory account of the mechanism that caused the formation of the ocean basins has yet been given. The traditional view supposes that the uppermantle of the earth behaves as a liquid when it is subjected to small forces for long periodsand// that differences in temperature under oceansand continents are sufficient to produce convection in the mantle of the earth with rising convection currents under the mid-ocean ridgesand sinking currents under the continents (传统观点,重点在mantle of the earth的liquid以及温度不同导致的convection上).Theoretically(理论上,期待转折在后面), this convection would carry the continental plates along as though they were on a conveyor belt and would provide the forces needed to produce the split that occurs along the ridge. This viewmay be correct(继续传统观点): it has the advantage that the currents are driven by temperature differences that themselves depend on the position of the continents. Such a back-coupling反馈, in which the position of the moving plate has an impact on theforces that move it, could produce complicated and varying motions.
  On the other hand, the theory is implausible(转了,传统不可行)because convection does not normally occur along lines and it certainly does not occur along lines broken by frequent offsets or changes in direction, as the ridge is. Also it is difficult to see how the theory applies to the plate between the Mid-Atlantic Ridge and the ridge in the Indian Ocean.(对抗传统观点的反例) This plate is growing on both sides, and since there is no intermediate trench, the two ridges must be moving apart. It would be odd if the rising convection currents kept exact pace with them. Analternative theory (新观点一)is that the sinking part of the plate, which is denser than the hotter surrounding mantle, pulls the rest of the plate after it. Again it is difficult to see how this applies to theridge in the South Atlantic, where neither theAfrican nor the American plate has a sinking part(新观点一反例).
     Another possibility (新观点二)is that the sinking plate cools theneighboring mantle and produces convectioncurrents that move the plates. This last theory is attractive (正向态度词,很好)because it gives some hope of explaining theenclosed seas, such as the Sea of Japan.(支持新观点二的例子1) These seas have atypical oceanic floor, except thatthe floor is overlaid by several kilometers ofsediment. Their floors have probably been sinkingfor long periods. It seems possible that a sinkingcurrent of cooled mantle material on the upper side of theplate might be the cause of such deepbasins. The enclosed seas are an importantfeature of the earth’s surface, and seriouslyrequire explanation in because, addition to theenclosed seas that are developing at present behindisland arcs, there are a number of older ones ofpossibly similar origin(支持新观点二的例子2), such as the Gulf ofMexico, the Black Sea, and perhaps the North Sea.
(1) According to the traditional view (传统观点,定位第一段)of the origin of the ocean basins, which of the following is sufficient to move the continental plates?
A. Increases in sedimentation on ocean floors
B. Spreading of ocean trenches
C. Movement of mid-ocean ridges
D. Sinking of ocean basins
E. Differences in temperature under oceans and continents
(原文:that differences in temperature under oceans and continents are sufficient to produce convection in the mantle of the earth with rising convection currents under the mid-ocean ridges and sinking currents under the continents )

(2) It can be inferred from the passage that, of the following, the deepest sediments would be found in the
A. Indian Ocean
B. Black Sea (是因为新观点二的deep basins吗?)
C. Mid-Atlantic
D. South Atlantic
E. Pacific

(3) The author refers to a "conveyor belt" in order to:
A. illustrate the effects of convection in the mantle (定位第一段传统观点,theoretically后面,原句是对传统观点的进一步解释)
B. show how temperature differences depend on the positions of the continents
C. demonstrate the linear nature of the Mid-Atlantic Ridge
D. describe the complicated motions made possible by back-coupling
E. account for the rising currents under certain mid-ocean ridges

(4) The author regards the traditional view of the origin of the oceans with
A. slight apprehension
B. absolute indifference
C. indiganant anger
D. complete disbelief
E. guarded skepticism (虽然说了implausible,但是前面用了那么多让步:theoretically,may be correct)

(5) According to thepassage, which of the following are separated by a plate that is growing on bothsides?
(A) The Pacific Ocean and the Sea of Japan
(B) The South Atlantic Ridge and the NorthSea Ridge
(C) The Gulf of Mexico and the South AtlanticRidge
(D) The Mid-Atlantic Ridge and the IndianOcean
Ridge (定位第二段,新观点提出前面的原话:this plate is growing on both sides)
(E) The Black Sea and the Sea of Japan

(6) Which of the following,if it could be demonstrated, would most support the traditional view ofocean formation?
(A) Convection usually occurs along lines. (先找原文中反对传统观点的地方,定位第二段,对传统观点反驳的原话:because convection does not normally occur along lines)
(B) The upper mantle behaves as a densesolid.
(C) Sedimentation occurs at a constant rate.
(D) Sinking plates cool the mantle.
(E) Island arcs surround enclosed seas.

(7) According to thepassage, the floor of the Black Sea can best be compared to a
(A) rapidly moving conveyor belt
(B) slowly settling foundation (black sea是新观点二,定位最后一段their floors have probably been sinking for long periods)
(C) rapidly expanding balloon
(D) violently erupting volcano
(E) slowly eroding mountain

(8) Which of the followingtitles would best describe the content of the passage?
(A) A Description of the Oceans of the World
(B) Several Theories of Ocean Basin Formation(除了这个,其他只提了某一个观点,都不全面)
(C) The Traditional View of the Oceans
(D) Convection and Ocean Currents
(E) Temperature Differences Among theOceans of
the World

叹口气,分析花了好长时间,考试的时候恐怕没这么多时间也没这么清晰的思路...汗啊..
阅读心得:要克服在第一次阅读时,把每句话都读懂的心情!什么最重要?结构和态度最重要!
                细节定位不会跑出同一段。



4.阅读小分队上金砖五国文章一篇,我的笔记:http://forum.chasedream.com/forum.php?mod=viewthread&tid=827938&page=6#pid18503730
很短,文章贴上来:
BRAZIL, Russia, India, China and South Africa recently concluded the fifth annual meeting of the countries known collectively as the BRICs. Or should that be the BRICS? The confusion arises from the fact that South Africa has sneaked into the group, which claims to represent the world’s emerging markets and act as a counterweight to the G8 and G20, which are dominated by rich-world economies.
The BRIC countries were the constituent members of an acronym coined by Jim O’Neill, then of Goldman Sachs, in 2001. Mr O’Neill was looking for a way to convey the fact that much of the world’s economic growth would soon come from Brazil, Russia, India and China. There was much debate about whether this grouping made sense: at the time Brazil’s growth seemed too sluggish to warrant inclusion; now Russia looks like it doesn’t deserve to be placed with the others. China has a much higher economic growth rate than the rest. Even so, the label proved so catchy that the foreign ministers of the BRIC countries decided to hold a summit in New York in 2006. What began as a hook for an investment bank's research note became a real political institution.
There was just one problem with the BRICs: no African countries were included. This was a little embarrassing. Overlooking Africa suggested that the continent was an economic irrelevance, good only for providing raw materials to the rest. It also cast doubt on the group’s claim to speak for the emerging world. Two African countries might have been candidates, Nigeria and South Africa. But only one would keep the acronym intact. And so, in 2010, the club of BRICs became the BRICS.
The strange etymology of the BRICS has real-world consequences. Though the inter-governmental meetings have not amounted to much yet, these countries do have ambitions to set up a joint investment bank. It is easier to reach agreements in small groups than in big ones. China and Brazil struck a currency-swap deal to facilitate trade at this year's meeting. South Africa has thus gained some real political advantage—at least until someone invents a better acronym.

Time 5: 2'34" 354words
acronym首字母缩略词
how the BRIC comes from, why North Africa joined as a BRICS
(又没有想明白的一句话:China and Brazil struck a currency-swap deal to facilitate trade at this year's meeting. South Africa has thus gained some real political advantage—at least until someone invents a better acronym.为什么说BRICS的语源很奇怪?..)
版主的回复:
(1). 先解释这个句子:
意为:中国和巴西已经签订了货币互换协议以促进今年的会议(此处的meeting是指金砖四国加了南非以后的“金砖五国”会议:即fifth annual meeting of the countries known collectively as the BRICs or BRICS). 南非也因此获得了一些真正的政治优势(至少在比BRICS这个更好的首字母缩略词出现之前是这样的。)(此处的政治优势是指第2段段尾提到的:What began as a hook for an investment bank's research note became a real political institution.)
*strike a deal=strike a bargain=to reach an agreement on a price or negotiation (for something).达成协议 eg. <They argued for a while and finally struck a bargain.> <They were unable to strike a bargain, so they ended their meeting.>
(2).为什么说BRICS的语源很奇怪?你是说这一段的句首吧“The strange etymology of the BRICS has real-world consequences.”?因为以前就是BRICs-金砖四国(代表了world’s economic growth
),那个s代表复数。但是因为没有任何非洲国家的加入,有些遗憾。而南非入选的原因也很奇怪:即原文提到的:
Two African countries might have been candidates, Nigeria and South Africa. But only one would keep the acronym intact. And so, in 2010, the club of BRICs became the BRICS.
看到了吧。就是因为南非的(South Africa)的首字母是S, 正好把BRICs凑成了BRICS. (这样既满足了1).找个国家和金砖四国有相似之处:represent the world’s emerging markets),2).非洲国家中的经济强国,3).首字母是S为佳(所以Nigeria不符合条件3)被刷掉了嘛)。
是不是这么看来,这个词源来的很奇怪呢?嗯。就是这个意思啦~



5.
很喜欢的一道语法比较题:
OG10-198. Although Napoleon's army entered Russia with far more supplies than they had in their previous campaigns, it had provisions for only twenty-four days.
A. they had in their previous campaigns
B. their previous campaigns had had
C. they had for any previous campaign
D. in their previous campaigns
E. for any previous campaign
----------------------------------------------

If than is followed by a clause referring to army, the subject of that clause must be singular (it). Furthermore, the verb of that clause will need to be in the past perfect form (had had) because it refers to a time before the simple past of entered. Finally, the preposition for is more precise than in because supplies are gathered for an upcoming campaign. Choice A and C incorrectly use the plural they and the simple past had. Moreover, A uses the less precise in. Choices D and E wisely dispense with the full clause and use a simple prepositional phrase. D, however, uses the imprecise in and the plural their. Only E, the best choice, avoids all the errors mentioned above.
消除一切歧义有木有!代词神马的最容易有歧义,消除了最痛快有木有~

6. Prep08
#2-206: For the first time in the modern era, non-Hispanic Whites are officially a minority in California, which amounts to a little less than half the population of the state, downfrom nearly three-quarters only a decade ago
A.which amounts to a little less than half the population of the state, down fromnearly three-quarters only a decade ago
B.which amounts to a little less than half the population of the state, down froma decade ago, when it was nearly three-quarters
C.and that amounts to a little less than half the population of the state, downfrom a decade ago, when they were nearly three-quarters
D.amounting to a little less than half the population of the state, down fromnearly three-quarters a decade ago
E.amounting to a little less than half the population of the state, down fromwhat it was a decade ago by nearly three-quarters
-----------------------------------------------
选D。E选项中down from what it was a decade ago比较对象不合理,应该是人口数与人口数比,而不是人口与时间;it无所指(网上有帖子说就近指,是不是the population of state,即使是显然也是不对的);what it was 很wordy and awkward;down from...by改变原句含义

8月18日(日)今天好像权志龙生日...我不是饭...网上说太多...
单词,阅读小分队,语法Prep08照旧,这三项成为日常内容,每日车轮复习,以后不表。
Prep08打算采取的方式为隔一天复习两天,记录易错题,连续两次记录下来的题目总结下来,反复熟悉。
今天的阅读小分队讲美国独立战争之前的一些法律,想到之前看的一些英文法律文件,可以再看,现代法律的始祖诶

http://forum.chasedream.com/forum.php?mod=viewthread&tid=870366&extra=page%3D1%26filter%3Dtypeid%26typeid%3D169&page=1

63篇第六篇分析,总觉得人文历史类反而比科技类难...
Passage 6 (6/63)问题解决型(与历史有关)
史实:有人物有时间——提出现象(如果后面讲原因——现象解释型;如果后面讲如何改变——问题解决型——本文)
In the eighteenth century, Japan’s feudal overlords, from the shogun将军 (shogun: n.<日>幕府时代的将军) to the humblest samurai, found themselves under financial stress. 总结大原因In part, this stress can be attributed to(原因) the overlords’ failure to adjust to a rapidly expanding economy, but(后面的原因更重要) the stress was also due to(原因) factors beyond the overlords’ control. Concentration of the samuraiin castle-towns had acted as a stimulus to trade. Commercial efficiency, in turn,had put temptations in the way of buyers刺激了购买欲. 第一个小原因:武士问题Since most samuraihad been reduced to idleness不用,闲置 by years of peace, encouraged to engage in scholarship and martialexercises or to perform administrative tasks that took little time, it is not surprising that*3B(小结论)(略带同情的正评价) theirtastes and habits grew expensive. 第二个小原因:收入增加跟不上支出增加Overlords’ income, despite(转折) the increase in rice production among their tenant farmers, failedto keep pace with their expenses. Although shortfalls in overlords’ incomeresulted almost as much from 玩忽职守laxity among their tax collectors*6(the nearly inevitable outcome of hereditaryoffice-holding*6C) as from 高生活水平theirhigher standards of living, 第三个小原因:灾难(转折后更重要的部分)a misfortune like a fire or flood, bringing anincrease in expenses or a drop in revenue, could put a domain in debt*5 to the city rice-brokers whohandled its finances. Once in debt, neither the individual samurai nor theshogun himself found it easy to recover.第一个大原因:支出的增加
第二个大原因:不能增加收入It was difficult forindividual samurai overlords to increase their income because the amount ofrice that farmers could be made to pay in taxes was not unlimited*5E有限制的, and since the income of Japan’s central government consisted inpart of taxes collected by the shogun from his huge domain, the government toowas constrained. 问题的解决Therefore, the Tokugawashoguns began to look to other sources for revenue. 第一种解决方案Cash profits fromgovernment-owned mines were already on the decline(-) because the most easily workeddeposits of silver and gold had been exhausted, although debasement of thecoinage had compensated for the loss. 第二种解决方案Opening up new farmland was a possibility, but most of what wassuitable had already been exploited and further reclamation was technically unfeasible(-). 第三种解决方案Direct taxation of the samurai themselves would be politically dangerous(-). 承上启下This left the shoguns only commerce as a potential source ofgovernment income.
Most of the country’s wealth*8B, or so itseemed, was finding its way into the hands of citymerchants*8. It appeared reasonable that they should contribute part ofthat revenue to ease the shogun’s burden of financing the state. A means ofobtaining such revenue was soon found bylevying forced loans(主题词), known as goyo-kin御用金; although these werenot taxes in the strict sense, since they were irregular in timing andarbitrary in amount, they were high in yield(+). Unfortunately(-), they pushed up prices*9A. Thus, regrettably*9,the Tokugawa shoguns’ search for solvency for the government made itincreasingly difficult for individual Japanese who lived on fixed stipends tomake ends meet.
1.     Thepassage is most probably an excerpt from主题题(内容性)
(A) an economichistory史实 of Japan
(B) the memoirs不是某个人的回忆录 of a samurai warrior
(C) a modern novel是史实,不是虚构的小说 abouteighteenth-century Japan
(D) an essaycontrasting Japanese feudalism with its Western没提到 counterpartA
(E) anintroduction to a collection of Japanese folktales不是民间故事
2.     Whichof the following financial situations is most analogousto the financial situation in which Japan’s Tokugawa shoguns found themselvesin the eighteenth century?类比题(还有similar toparallel也是类比题:抓住本质特征)错了
(A) A smallbusiness borrows heavily to invest in new equipment, but is able to pay off itsdebt early when it is awarded a lucrative government contract.
(B) Firedestroys a small business, but insurance covers the cost of rebuilding.
(C) A smallbusiness is turned down for a loan at a local bank because the owners have nocredit history.
(D) A smallbusiness has to struggle to meetoperating expenses when its profits decrease. 收入跟不上支出增加D
(E) A smallbusiness is able to cut back sharply on spending through greater commercialefficiency and thereby compensate for a loss of revenue.
3.     Whichof the following best describes the attitude of the author toward the samuraidiscussed in lines 11-16?态度题
(A) Warmlyapproving
(B) Mildly sympathetic*3B略带同情的正评价
(C) Bitterlydisappointed
(D) HarshlydisdainfulB
(E) Profoundlyshocked
4.     Accordingto the passage, the major reason for thefinancial problems experienced by Japan’s feudal overlords in the eighteenthcentury was that(送分题)
(A) spendinghad outdistanced income收入跟不上支出增加
(B) trade hadfallen off
(C) profitsfrom mining had declined
(D) the coinagehad been sharply debasedA
(E) the samuraihad concentrated in castle-towns
5.     Thepassage implies that individual samurai didnot find it easy to recover from debt*5(第一段结尾,用第二段开头做题) for which of the following reasons?直接事实题
(A)Agricultural production had increased.
(B) Taxes wereirregular in timing and arbitrary in amount.
(C) TheJapanese government had failed to adjust to the needs of a changing economy.
(D) The domainsof samurai overlords were becoming smaller and poorer as government revenuesincreased.E
(E) There was alimit*5E to the amount in taxes that farmerscould be made to pay.
6.     Thepassage suggests that, in eighteenth-century Japan, the office of tax collector*620行左右)(括号——本题,引号,破折号内容会出题)错了
(A) was asource of personal profit to the officeholder
(B) wasregarded with derision by many Japanese
(C) remained within families*6C职位世袭
(D) existedonly in castle-townsC
(E) took upmost of the officeholder’s time
7.     Whichof the following could best be substituted for the word “This”(放在三个已经不行的方法之后) in line47 without changing the meaning of the passage?错了
(A) The searchof Japan’s Tokugawa shoguns for solvency(干扰选项)
(B) Theimportance of commerce in feudal Japan
(C) Theunfairness of the tax structure in eighteenth century Japan
(D) Thedifficulty of increasing government income by other means其他方法不行D
(E) Thedifficulty experienced by both individual samurai and the shogun himself inextricating themselves from debt
8.     Thepassage implies that which of the following was theprimary reason why the Tokugawa shoguns turnedto city merchants*8(最后一段) for help in financing the state?错了
(A) A series ofcostly wars had depleted the national treasury.
(B) Most of the country’s wealth*8B appeared tobe in city merchants’ hands.
(C) Japan hadsuffered a series of economic reversals due to natural disasters such asfloods.
(D) Themerchants were already heavily indebted to the shoguns.B
(E) Furtherreclamation of land would not have been economically advantageous.
9.     Accordingto the passage, the actions of the Tokugawa shoguns in their search forsolvency for the government were regrettable*9(60) because those actions直接事实题
(A) raised thecost of living by pushing up prices*9A
(B) resulted inthe exhaustion of the most easily worked deposits of silver and gold
(C) were farlower in yield than had originally been anticipated
(D) did notsucceed in reducing government spendingA
(E) acted as adeterrent to trade

iamyingjie给我的复习建议,贴在这里鼓励一下自己。
这个就是温故而知新的道理咯。所以平时要多加强复习力度,把这种反复强化、加深理解的过程变成能力的提升,这样考场上就不会怯场了。再读一遍是绝对必要的。2-3遍也不算多余,只要你每一遍都是认真的。但是第一遍的时候一定要最认真,要计时,并告诉自己这个第一遍就是模拟考场的状态,必须认真对待,不要想着还有第二次的机会。
板凳
 楼主| 发表于 2013-8-21 15:28:28 | 只看该作者
8月21日(三)
昨天休息一天,今天看阅读小分队都觉得效率高了。
阅读寂静1-10


原文考古:
(1) Covergence theory - Wiki
The idea of convergence in economics (also sometimes known as the catch-up effect) is the hypothesis that poorer economies' per capita incomes will tend to grow at faster rates than richer economies. As a result, all economies should eventually converge in terms of per capita income. Developing countries have the potential to grow at a faster rate than developed countries because diminishing returns (in particular, to capital) are not as strong as in capital-rich countries. Furthermore, poorer countries can replicate the production methods, technologies, and institutions of developed countries.
In economic growth literature the term "convergence" can have two meanings. The first kind (sometimes called "sigma-convergence") refers to a reduction in the dispersion散布,离差 of levels of income across economies. "Beta-convergence" on the other hand, occurs when poor economies grow faster than rich ones. Economists say that there is "conditional beta-convergence" when economies experience "beta-convergence" but conditional on other variables being held constant. They say that "conditional beta-convergence" exists when the growth rate of an economy declines as it approaches its steady state.
Limitation
The fact that a country is poor does not guarantee that catch-up growth will be achieved. Moses Abramovitz emphasised the need for 'Social Capabilities' to benefit from catch-up growth. These include an ability to absorb new technology, attract capital and participate in global markets. According to Abramovitz, these prerequisites must be in place in an economy before catch-up growth can occur, and explain why there is still divergence in the world today.
The theory also assumes that technology is freely traded and available to developing countries that are attempting to catch-up. Capital that is expensive or unavailable to these economies can also prevent catch-up growth from occurring, especially given that capital is scarce in these countries. This often traps countries in a low-efficiency cycle whereby the most efficient technology is too expensive to be acquired. The differences in productivity techniques is what separates the leading developed nations from the following developed nations, but by a margin narrow enough to give the following nations an opportunity to catch-up. This process of catch-up continues as long as the following nations have something to learn from the leading nations, and will only cease when the knowledge discrepancy between the leading and following nations becomes very small and eventually exhausted.
According to professor Jeffrey Sachs, convergence is not occurring everywhere because of the closed economic policy of some developing countries, which could be solved through free trade and openness. In a study of 111 countries in 1970 - 1989, Sachs and Andrew Warner concluded that the industrialized countries had a growth of 2.3%/year/capita, open economy developing countries 4.5% and closed economy developing countries only 2%.
Robert Lucas stated the «Lucas Paradox» which is the observation that capital is not flowing from developed countries to developing countries despite the fact that developing countries have lower levels of capital per worker. This statement, however, has received recently serious objections
Examples
There are many examples of countries which have converged with developed countries which validate the catch-up theory. In the 1960s and 1970s the East Asian Tigers rapidly converged with developed economies. These include Singapore, Hong Kong, South Korea and Taiwan - all of which are today considered developed countries or cities. In the post-war period (1945–1960) examples include West Germany, France and Japan, which were able to quickly regain their prewar status by replacing capital that was lost during World War II.
Some economists criticise the theory, stating that endogenous factors, such as government policy, are much more influential in economic growth than exogenous factors. For example, Alexander Gerschenkron states that governments can substitute for missing prerequisites to trigger catch-up growth. A hypothesis by economic historians Kenneth Sokoloff and Stanley Engermansuggested that factor endowments are a central determinant of structural inequality that impedes institutional development in some countries. Sokoloff and Engerman proposed that in the 19th century, countries such as Brazil and Cuba with rich factor endowments捐赠,天资 such as soil and climate are predisposed to a guarded franchise with limited institutional growth. Land that is suitable for sugar and coffee such as Cuba experienced economies of scale from the establishment of plantation that in turn created the small elite families with vested interest in guarded franchise. The exogenous外生的,外因的 suitability of land for wheat versus sugar determines the growth rate for many countries. Therefore, countries with land that is suitable for sugar converge with other countries that also have land that is suitable for growing sugar.
Sokoloff and Engerman explained this convergence in their article "History Lessons: Institutions, Factor Endowments, and Paths of Development in the New World." They explained that the United States and Canada started out as two of the poorest colonies in the New World but grew faster than other countries as a result of their soil qualities. They argued that the United States and Canada had land suitable for growing wheat which meant that they had small scale farming, since wheat does not benefit from economies of scale, and this led to a relatively equal distribution of wealth and political power enabling the population to vote for broad public education. This differentiated them from countries such as Cuba that had land suitable for growing sugar and coffee. Such countries did benefit from economies of scale and so had large plantation agriculture with slave labor, large income and class inequalities, and limited voting rights. This difference in political power led to little spending on the establishment of institutions such as public schools and slowed down their progress. As a result, countries with relative equality and access to public education grew faster and were able to converge on countries with inequality and limited education.
(2)黑人妇女
In many ways, Mary Jane McLeod Bethune's life was representative of the lives of many African American women of her time: she was deeply grounded in religion and family, and intensely committed to racial advancement. Yet, Bethune became one of the most important African American women in American political history. She came to occupy a prominent place among a select group of black men and women designated as “race leaders”—men and women who devoted their lives to advancing African American equality. They became the public voice of the voiceless masses, speaking of the collective identity of people of color and arguing for equal social, economic, and political rights. Bethune was certainly a pivotal member of this group as her efforts advanced equal opportunity for black Americans on all levels for nearly half a century. Yet, Bethune distinguished herself from other race leaders by steadfastly incorporating the struggle for gender equality within her efforts for black equality. By advocating and training black women for visible and increasing public leadership roles, she ensured an expanding role for African American women in the formal political realm. She believed this would automatically lead to advancement for the entire race, as black women then were more inclined than black men to use public positions for group advancement. Bethune's exposure to strong, independent female role models allowed her to develop her unwavering belief in the primary responsibility of black women for sustaining the race. Her grandmother, mother, and female teachers demonstrated how black women who embraced “a larger appreciation for good citizenship, cleanliness, beauty, thoughtfulness” could lead African Americans as “the mothers of the race, the homemakers and spiritual guides.” Bethune believed African American women had an obligation to understand these responsibilities and use their status to fight for equality. She publicly endorsed the notion of women's higher moral capacity, recognized the important contribution women could make to racial uplift, and continually worked to expand women's roles toward that end. Bethune was a truly multifaceted and multidimensional race woman. She fought on a variety of levels and used multiple outlets—education, government, and women's associations—in her quest for a more just society. Some black women leaders before her gained more recognition than she achieved in her lifetime, but none before her, and few afterwards, were more effective in developing women's leadership for the cause of racial justice.

Despite her multiple political activities, Bethune has not been recognized as a black political leader. This is attributable in large part to the traditional definition of political activity used by many historians and political scientists: political activity encompasses the actions of individually elected officials and the workings of government. It also rests upon a conventionally accepted and gender-biased idea of a leader as a “spokesman, ” and of politics as voting, electioneering, and office holding. This traditional research defines women's political participation as atypical, seeing women as inadequately socialized into the political process. It ties women's political activism to their social roles as wives and mothers. Women such as Bethune who entered the public arena and fought for substantive reform while remaining grounded in networks of kin, church, and community were left out of political history. As feminist historians have become more interested in political history, they have worked to redefine politics as any “activity [that] includes all community work which is oriented to change through multifaceted goals including service, support, public education and advocacy. Political orientation [is adapted] to changing the public agenda through planned and implemented actions.” Empowerment is an important part of women's politicization and begins when women “change their ideas about the causes of their powerlessness, when they recognize the systematic forces that oppress them, and when they act to change the conditions of their lives. Using this definition, black women who worked through voluntary associations and community organizations became political leaders because they brought particular issues to the attention of politicians and the public. They fought for equal opportunity for African American men and women at a time when America had neither the will nor the desire to make a commitment to racial or sexual equality. Bethune is one such woman who deserves recognition as a political leader based upon the depth and breadth of her political activities.

However, even the few historians who have given passing attention to Bethune's political accomplishments have misinterpreted the means, techniques, and actions she employed in pursuing equality. When examined individually, the choices she made throughout her lifetime often appear contradictory, unless we understand that Bethune had one foot in the nineteenth century and one in the twentieth. She was a transitional figure. Initially grounded in the nineteenth-century belief that advancement would come through changing individual behavior, Bethune in the twentieth century quickly recognized that inequality was deeply rooted in American institutions. She began to see that the focal point for African Americans should no longer be on changing individual attitudes and behaviors, but rather on changing social, economic, and political institutions that shaped collective opinions. She worked diligently to transform local community groups into political power bases and promoted the formation of a national coalition that would work to alter social, economic, and political institutions. In these efforts, she used two conceptually distinct levels of activism. In some instances, Bethune based her activism on informal political activities that were distinctly nonconfrontational and designed to quietly undermine racial and gender stereotypes. Yet, when dealing with egregious incidents involving institutional inequality, Bethune often engaged in overtly formal political action that publicly challenged the basic principles of the American democratic system. She astutely gauged her activism to fit the particular circumstances. And no matter which course Bethune decided to pursue, she sought a peaceful, yet political, means to achieve social, economic, and political justice.

Mary Jane McLeod Bethune was born on July 10, 1875, in Mayesville, South Carolina, the fifteenth of seventeen children born to Samuel and Patsy McLeod. She was educated at the local missionary school, then received scholarships from a Quaker dressmaker that enabled her to attend Scotia Seminary and Moody Bible Institute. Between 1895 and 1903, she taught at a number of small missionary schools throughout the South, including Haine's Institute in Augusta, Georgia. In 1898, she met and married Albertus Bethune and in 1899 gave birth to her only child, Albert McLeod Bethune. In 1904, she traveled to Daytona Beach, Florida, where she established the Daytona Educational and Industrial Institute for Negro Girls capitalized with her personal savings of $1.50. By 1912, the school offered a liberal arts high school curriculum and employed nine full-time teachers. In 1923, Daytona Institute merged with Cookman Institute, becoming the coeducational Bethune-Cookman College. Bethune-Cookman became the first fully accredited four-year college for blacks in Florida. Bethune served as its president until 1942.

In addition to establishing and operating Daytona Institute, Bethune served in a variety of roles in a diverse array of commissions and organizations. She was recognized as an expert on black education and was an active member of the National Commission for Child Welfare under Presidents Coolidge and Hoover. She was also president of state, regional, and national women's clubs including the Florida State Federation of Colored Women's Clubs, the Southeastern Federation of Colored Women's Clubs, and the National Association of Colored Women's Clubs (NACW). In 1927, she met Eleanor Roosevelt through her position as president of the National Association of Colored Women's Clubs and by 1935 their growing friendship led to her appointment as director of the Office of Minority Affairs in the National Youth Administration, the first federal office created for a black woman. In the same year, Bethune organized the National Council of Negro Women (NCNW), an umbrella organization designed to give black women political visibility and access to political power on the national level. Bethune continued to serve African Americans through a federal appointment in 1942 as special assistant to the Secretary of War for Selection of Candidates for the first Officers Candidate School for WACS. In 1945, President Harry Truman named Bethune to his Civil Rights Commission and as the only African American woman consultant to the San Francisco Conference to draw up the charter for the United Nations.

During her lifetime, Bethune received numerous awards, eight honorary degrees, and held affiliations with at least seventy-five organizations, including the General Conference of the Methodist Church, the Women's Army for National Defense, National Commission on Christian Education, American Women's Volunteer Service, Southern Conference Education Fund, American Mother's Committee, Council of Church Women, Social Service Commission of the Methodist Church, Americans for Democratic Action, National Civil Liberties Union, First Daytona Beach Housing Authority, American Council on African Education, Inc., National Committee on Atomic Information, Good Neighbor Association, Daytona Beach, Florida, the International Longfellow Society, National Council of Women of the United States, vice president of the Southern Conference for Human Welfare, a director of the Afro-American Life Insurance Company, and the vice president of the Central Life Insurance Company. She founded the Mary McLeod Bethune Foundation, and Bethune Beach, Inc. Bethune received honorary degrees from Wilberforce University, South Carolina State College, Lincoln University, Tuskegee University, Howard University, Bennett College, West Virginia State College, and Rollins College. She was rewarded for her service to the race and her commitment to American democracy by receiving the Spingarn Medal, the Thomas Jefferson Award, the First Annual Youth City Award, the Haitian Medal of Honor, and the Star of Africa. In addition to her service work, Bethune made many literary contributions, including one chapter in What the Negro Wants, one chapter in Spiritual Autobiography,a weekly column in the Chicago Defender and the Pittsburgh Courier, and numerous articles in magazines and periodicals including editorials for Afraamerican Woman and Women United, the official publications of the National Council of Negro Women. Journalist Ida Tarbell named her among the fifty women regarded as having done the most for the welfare of the United States. Bethune died on May 18, 1955.

8月22日(四)
2400多个单词已经全过完一遍,开始反复温习。单词卡片继续增加。
总结代词部分语法要点,并对照千行梳理OG上的代
词部分题目。
小规模开始做Prep07的题目,不纠结于上面的解释。


1. 重新读阅读小分队的一篇文史哲,有不同的体会。我的笔记:http://forum.chasedream.com/forum.php?mod=viewthread&tid=869147&page=8#pid18491257
Time 2: 2'47" 342words
resent怨恨,厌恶
The parliament started to charge duties on paper, lead and tea, and depolyed soliders in port. The conflict between soliders and citizens became severe. Finally, the "Boston Massacre" broke out.
Time 3: 2'06" 264words
American colonist capitalized on this event to gain victory.
(一开始没读懂的句子:More recent scholars, however, have found evidence of a more discerning Boston public that was appreciative of British restraint and disapproving of provocative mob actions. Evidence of the latter view was found in the relative quiet that descended on the community after the funeral.)
重新看了Time2和Time3这一篇,文章较中立的描述了士兵和市民的冲突,英国士兵遵守章程,受到袭击也没有开枪,船上的炮火由于误会而射向民众:An argument broke out between the soldier and a local merchant, who was struck with the butt of a musket步枪,毛瑟枪 during the confrontation. A crowd assembled quickly and began pelting投掷,盛怒的 the sentry哨兵 with a variety of materials — stones, oyster shells, ice, and chunks of coal. Tensions were further heightened when the bells of the city’s churches began to toll, the traditional means of summoning召唤,振作 help in fighting fires. Reinforcements under Captain Thomas Preston were rushed in to relieve the beleaguered围困的,包围的 sentry. The mob taunted奚落 the soldiers, daring them to fire, while remaining somewhat secure in the widely held knowledge that the soldiers could not discharge their weapons within the city without prior authorization from a civil magistrate地方法官,文职推事.
所以才会有上面那句一开始没有读懂的句子。美国殖民者利用这个事件煽动民众情绪,但历史学家也发现,有民众并不完全同意攻击英国士兵的行为。也有殖民者站出来维护英国士兵:Later, 35-year-old John Adams risked the disapproval of his friends and neighbors by defending the British soldiers in a highly publicized trial.

2. OG及Prep07
OG23: In a previous design, the weight of the discus used in track competition is concentrated in a metal center, but now it is lined with lead around the perimeter, thereby improving stability in flight and resulting in longer throws.
A. In a previous design, the weight of the discus used in track competition is concentrated in a metal center, but now it is
B. According to a previous design, the weight of the discus used in track competition was concentrated in a metal center, but now it is
C. Once designed with its weight concentrated in a metal center, the discus used in track competition is now
D. The discus used in track competition, once designed with its weight concentrated in a metal center, but now
E. The discus used in track competition was once designed having its weight concentrated in a metal center and now
---------------------------------------------------------
A中的previous和now都是时态的时间指示词,所以第一个is应用was;代词的指代问题。B中的according to,Ron也认为有问题:"according to..." doesn't make sense in this context. that's a phrase you use when you name the person or source from which some information/opinion/etc. comes.
e.g.according to GMAC, the GMAT exam is designed to measure test-takers' reasoning abilities, not their knowledge of particular content.

E选项存在习语的使用错误:design的正确搭配是:be designed to do;designed doing不对

OG101:Heavy commitment by an executive to a course of action,especially if it has worked well in the past, makes it likely to miss signs ofincipient trouble or misinterpret them when they do appear.
A.Heavy commitment by an executive to a course of action, especially if it hasworked well in the past, makes it likely to miss signs of incipient trouble ormisinterpret them when they do appear.
B.An executive who is heavily committed to a course of action, especially onethat worked well in the past, makes missing signs of incipient trouble ormisinterpreting ones likely when they do appear.
C.An executive who is heavily committed to a course of action is likely to missor misinterpret signs of incipient trouble when they do appear, especially ifit has worked well in the past.
D.Executives’ being heavily committed to a course of action, especially if it hasworked well in the past, makes them likely to miss signs of incipient troubleor misinterpreting them when they do appear.
E.Being heavily committed to a course of action, especially one that has workedwell in the past, is likely to make an executive miss signs of incipienttrouble or misinterpret them when they do appear.
------------------------------------------------------------
思路:E-GMAT的详细解释:
Lets begin the solution:
Step 1 - Read the original sentence and understand the meaning.

Heavy commitment by an executive to a course of action, especially if it has worried well in the past, makes it likely to miss signs of incipient trouble or misinterpret them when they do appear.
(Note that even though you were all able to eliminate choice A, we still need to review this choice to understand the meaning that the correct choice is intended to communicate).
1. Sentence talks about an executive who is heavily committed to a course of action
2. This course of action has worked well in the past
3. Because of this heavy commitment, the executive is likely to miss the signs of trouble when they appear.
Step 2 - We will understand the errors in this sentence:

Pronoun Error - ..makes it likely to…- “IT” has no clear antecedent. The sentence must specify clearly that executive is likely to miss the signs…
Thus, choice A is eliminated

Step 3 - Process of Elimination or Choice Analysis
Choice B
An executive who is heavily committed to a course of action, especially one that worked well in the past, makes missing signs of incipient trouble or misinterpreting ones likely when they do appear.

This choice changes the intended meaning of the sentence. Here is what this sentence communicates:
1. Executive is heavily committed to a course of action - Same as Intended Meaning
2. Executive makes missing signs of trouble likely - Different from Intended Meaning  

Thus, choice B states that executives makes the missing of signs likely, whereas, the intended meaning is that the heavy commitment to the course of action makes missing signs likely.
Choice C
An executive who is heavily committed to a course of action is likely to miss or misinterpret signs of incipient trouble when they do appear, especially if it has worked well in the past.
This choice does not distort the original meaning of the sentence. However, from this sentence it is not clear as to what has worked well in the past. Thus, this sentence has pronoun reference error for “it”
Choice E
Being heavily committed to a course of action, especially one that has worked well in the past, is likely to make an executive miss signs of incipient trouble or misinterpret them when they do appear.
This choice maintains the intended meaning:
1. Action of being heavily committed to a course of action causes the effect.
2. The effect is that the executive misses signs of trouble
Note here that the phrase “being heavily committed to a course of action” is the subject for the verb “is”.


Prep07-4: Nikola Tesla, the inventor of alternating current, because he was excited with the prospects of harnessing Niagara Falls for producing electric power, he predicted in the mid-1890's that electricity generated at Niagara would one day power the streetcars of London and the streetlights of Paris.
A. Nikola Tesla, the inventor of alternating current, because he was excited with the prospects of harnessing Niagara Falls for producing electric power, he
B. The prospects of harnessing Niagara Falls to produce electric power was exciting to Nikola Tesla, the inventor of alternating current, and so he
C. Excited about the prospects of harnessing Niagara Falls to produce electric power, Nikola Tesla, the inventor of alternating current,
D. Nikola Tesla, the inventor of alternating current, excited about the prospects of harnessing Niagara Falls for the production of electric power and
E. The inventor of alternating current, excited with the prospects of harnessing Niagara Falls for producing of electric power, Nikola Tesla
---------------------------------------------------
选C。D选项中for doing结构表示目的不及to do 有效;excited about...是形容词结构,如果要与predicted并列作谓语,需要加系动词be,否则结构不完整。

3.曼哈顿上try的固定用法
RIGHT:They WILL TRY TO BUILD a company(=intent or purpose)
SUSPECT:We TRIED BREAKING the door down.(=experiment)
WRONG:They WILL TRY AND BUILD a company.
               They WILL TRY THAT THEY BUILD a company.

8月23日(五)
语法,总结修饰语部分
阅读JJ 11-20

阅读JJ原文考古
诗歌
The first installment of Testimony was published in 1934 by the Objectivist Press, which had been started several years earlier to promote the views of poets including William Carlos Williams, Louis Zuk of sky, George Oppen, and Reznikoff himself .They were believers in Objectivism, a short-lived but still influential offshoot of poetic Modernism, the early 20th-century assault by T. S. Eliot,Ezra Pound, and others on the Enlightenment-influenced poetics of their predecessors. For the Objectivists, the poem was an object, not a report by the poet of what he or she thought or felt. They rejected the emphasis by19th-century Romantic poets like Wordsworth and Shelley on the poet's subjective experience of transcendent meaning as depicted through metaphor and symbol. (The title and opening line of Wordsworth's well-known poem about daffodils, "I wandered lonely as a cloud," is a good example of the tendencies that the Objectivists judged artificial and misleading.) The Objectivists believed that feeling and emotion should come through the choice of details and the sound and appearance of words on the page.
Reznikoff continued to work on Testimony throughout his life. In the 1960s, he published two new volumes (the first drawn from judicial opinions of 1885-1890, thesecond from opinions of 1891-1900); two additional volumes (1901-1910 and1911-1915) were published after his death. In each of the later volumes ,Reznikoff revised his art, reshaping the documentary material into syncopated lines of poetry.
The Negro was dead/when the doctors examined him," a characteristic poem begins: They found upon his belly bruises: he died, the doctors said, of peritonitis.
While the shift in form draws even more attention to the language (as in the isolation of "bruises" in the lines just quoted), the later editions employ thes ame third-person perspective, looking to the objective language of a judicial opinion, the words as words, rather than subjective experience or metaphor, for the emotional intensity of the poem. With its use of judicial opinions as the raw material of poetry, Testimony radically undercuts the traditional assumption that the poet works in a private sphere that is somehow separate from the pressures and pulls of the public domain. Not only is the poem an object, but it is an object taken from the workaday world that poets traditionally have viewed as unsuitable for poetry. Testimony never lets us forget that it is judicial opinions the poet is expounding.
Reznikoff's most important innovation and chief legacy to subsequent poets was this use of social speech, the public language of lawyers, to further the Modernist project of drawing attention to the linguistic qualities of a poem. By juxtaposing the descriptions of fact—the underlying story—of one case after another, he created an emotionally powerful collage from the apparently impersonal language of judicial opinions, a collage that chronicles America's struggle with slavery and its emergence as a commercial and industrial power.


8月24日(六)
OG上修饰语部分对应题目
新东方语法笔记,虽然有些话有点绝对,但有些点总结的还是有点帮助
小规模Prep07,前面44道题错了14道............


1.OG
OG-10:Carnivorous mammals can endure what would otherwise be lethal levels of body heat because they have a heat-exchange network which kept the brain from getting too hot.
A. which kept
B. that keeps
C. which has kept
D. that has been keeping
E. having kept
-------------------------------------------
选B,时态,限制性定从引导词都使用正确。
其中E选项having done是否能修饰名词network,看国外instructor给出的解释:
I will have to double-check, but I am nearly certain "Having + past participle" will only be a noun modifier when that noun is the subject of a clause.
论坛上补充的知识点:

1. having done 和 having been done都不是动名词结构,不能充当名词成分,所以不能做主语或宾语等名词可以充当的成分
2.having done 短语中所表达的内容发生在主句动作之前(就像neuroscientist那题)
present perfect所表达的情况(Ron):

the present perfect gives the impression that something has happened in the past, in a way that
* has persisted up to the present,
* still affects the present even though it's over, OR
* is over, but could happen again.

OG-30: For members of the seventeenth-century Ashamti nation in Africa, animal-hide shields with wooden frames were essential items of military equipment, a method to protect warriors against enemy arrows and spears.
A. a method to protect
B. as a method protecting
C. protecting
D. as a protection of
E. to protect
----------------------------------------------
选C。首先对于A和B,it is awkward and inaccurate to describe items themselves as a method。
关于to do的点:
Shields were essential items of military equipment, protecting warriors against arrows and spears.
“表达功能的话用doing更为合适” 这里是表示shields的功能

我另起炉灶的"to protect"的句子:
Members of Ashanti nation developed shields as essential items of military equipment, to protect warriors against arrows and spears.
“to do总是表示某一次具体的应用或目的” 这个句子是表示人们(members of Ashanti nation)这么做的目的

第30题,我觉得是说,这里"to protect"不能作为adverbial modifier,因此就不能用刚才说到的"comma + infinitive"这个adverbial modifier的结构。
为什么不能作adverbial modifier? 因为adverbial modifier是修饰的整个sentence/clause的意思.
这个句子主干:
Shields were essential items of military equipment.
所以"to protect"和这个句子不搭调。

如果你非要用逗号后面加"to protect",那我造这个句子:
Members of Ashanti nation developed shields as essential items of military equipment, to protect ....
你看,这个句子的主干,是:
Members developed shields.
Question: why did they develop shields?  Answer: To protect ....
Because "to protect ..." answers the "why" question, it is regarded as an adverbial modifier.

判断能否作为adverbial modifier,有个简单的办法,就是像我上面那样提问,看能否回答 the how/why/where/when 的问题.
当你这么检验的时候,你就会发现
"to protect" CANNOT answer the how/why/where/when questions for "Shields were essential items of military equipment".

综上,我的意思是,按原句的写法,"to protect"不能作为adverbial modifier,因为意思不搭调。而按我改动之后的写法才可以。
但是,"to protect"可以作为noun modifier修饰前面的noun "equipment",因为"equipment to protect" makes perfect sense!
在这种情况下,"to protect"前面就不能有逗号。所以OG会说:
the infinitive to protect would work if it were not preceded by a comma

OG-38: In 1850, Lucretia Mott published her Discourse on Women, arguing in a treatise for women to have equal political and legal rights and for changes in the married women's property laws.
A. arguing in a treatise for women to have equal politial and legal rights
B. arguing in a treatise for equal political and legal rights for women
C. a treatise that advocates women's equal political and legal rights
D. a treatise advocating women's equal political and legal rights
E. a treatise that argued for equal politial and legal rights for women
----------------------------------------------
选E。其中A和B除了用arguing..修饰published很奇怪外,是可以用更好的方法排除的(beat the GMAT instructor):
In A, it is unclear whether for women and for changes are adjectives modifying a treatise (a treatise for women...and for changes) or adverbs modifying arguing (arguing...for women...and for changes). It must be clear what a modifier is modifying. Eliminate A.
In B, it is unclear whether for equal political and legal rights for women and for changes are adjectives modifying a treatise (a treatise for equal political and legal rights for women and for changes) or adverbs modifying arguing (arguing...for equal political and legal rights for women and for changes). It must be clear what a modifier is modifying. Eliminate B.
关于这题为什么同位语会比ving清晰的原因(baby姐):
提炼一下就是这个样子:
LM published A, arguing in B for ......

然后就可以看出逻辑意思上的缺陷——并没有说清楚A和B是什么逻辑关系。尽管v-ing结构表伴随,但我们不知道是否B属于A的一部分?是否B=A?或者A和B就是两个没有交集的东东(LM同时做了这两件事)?
正确答案是absolute phrase结构,明确说明了B=A。
关于 that+verb和ing的一些细微区别(来自baby姐的canons shooting chickens):

还有,不是说当分词和that从句修饰一个名词时,如果从句有表达分词所不能表达出的逻辑含义择选从句,否则选分词更简洁吗,那在下面这一题里为什么不选A而选E呢?
According to United States Air Force officials, a cannon shooting dead chickens at airplanes has proved helpful to demonstrate what kind of damage can result when jets fly into a flock of large birds.
(A) shooting dead chickens at airplanes has proved helpful to demonstrate
(B) shooting dead chickens at airplanes has proved itself helpful as a demonstration of
(C) shooting dead chickens at airplanes proves itself helpful as demonstrating
(D) that shoots dead chickens at airplanes proves itself helpful to demonstrate
(E) that shoots dead chickens at airplanes has proved helpful in demonstrating

baby姐:
a cannon that is shooting dead chickens at airplanes has proved helpful
时态上就不make sense! 并不是说”has proved helpful”的背景画面是 a cannon shooting dead chickens.

下次要检验 “a noun + v-ing modifier” 是否make sense,我就插入”that be”或者”who be”,注意看时态是否make sense。比如这题就变成
a cannon that is shooting dead chickens at airplanes has proved helpful

Mitch:
In the SC above, a cannon that shoots is better than a cannon shooting. The -ing in shooting suggests continuous action, implying that the cannon is continuously shooting at chickens. Do we really think that the cannon is shooting at chickens 24 hours a day? The result would be a lot of dead chickens.

niksworth:
Coming to this question, the word shooting in a cannon shooting dead chickens, is a verbal. It is a participle which modifies cannon.
However, it is important to note that the participle has been used in the form of a reduced relative clause (i.e. a relative clause not marked by the use of an overt complementizer such as that)
If we modify the statement by including the relative pronoun, we immediately see the problem inherent in using the participle form here. - a cannon that is shooting dead chickens at airplanes has proved helpful. This shows that the use of participle form is not valid. This error is corrected in option E.(左下续)


OG-48: In 1713, Alexander Pope began his translation of the Iliad, a work that, taking hime seven years until completion, and that literary critic Samuel Johnson, Pope's contemporary, pronounced the greatest translation in any language.
A. his translation of the Iliad, a work that, taking him seven years until completion, and that literary critic Samuel Johnson, Pope's contemporary, pronounced
B. his translation of the Iliad, a work that took him seven years to complete and that literary critic Samuel Johnson, Pope's contemporary, pronounced
C. his translation of the Iliad, a work that had taken seven years to complete and that literary critic Samuel Johnson, Pope's contemporary, pronounced it as
D. translating the Iliad, a work that took seven years until completion and that literary critic Samuel Johnson, Pope's contemporary, pronounced it as
E. translating the Iliad, a work that had taken seven years to complete and literary critic Samuel Johnson, Pope's contemporary, pronounced it
---------------------------------------------------
选B,两个that,第一个that在从句中作主语,第二个that在从句中作宾语,所以一定要补出。
另外关于这个until completion和 to complete的区别(论坛讨论):
在manhattan上面我是没有找到关于take的习语用法,姑且认为两个都符合习语然后来分析一下

在manhattan的preferred rule里面首先有动词优于名词的rule,这里我觉得until completion不能表达出作者是complete的主语(maybe作者有翻译,然后最后是其他人代替了作者翻译最后的部分),所以明显没有complete来得好,至少在意思的传达上没有正确选项的动作主体来得清晰;这题主要考的还是逻辑和结构上的平行
CD上的观点:
正确idiom是it take sb sometime to do sth


OG-71:Unlike the original National Museum of Science and Technology in Italy, where the models are encased in glass or operated only by staff memebers, the Virtual Leonardo Project, an online version of the museum, encourages visitors to "touch" each exhibit, which thereby activates the animated functions of the piece.
A. exhibit, which thereby activates
B. exhibit, in turn and activation of
C. exhibit, and it will activate
D. exhibit and thereby activate
E. exhibit which, as a result, activates
--------------------------------------------------
选D。其中B选项OG给出的解释:In turn an activation...seems to be the subject of a new clause, but it has no verb, so the sentence is incomplete.
摘自PREP08的语法笔记(关于同位语的用法): 同位语
同位语结构的形式:  (参考白勇语法)
1) 名词性同位语, 名词解释名词:
n. , n. ; n. , a/an + n.或  a/an + n. , n.

2) that引导同位语结构:
抽象名词(evidence, result, principle等等) + that从句, that从句对抽象名词进行具体化解释

3) 概括性同位语结构:  
用一个概括性的名词去概括前面的解释对象;短语/句子, a program/ an advance/ a practice that…  

4) 名词重复性同位语结构:
n. , n.(重复所修饰的名词) + that…或  n. , the same n.(重复所修饰的名词) that…
参考OG12 126题对于in turn的解释:

In turn  is used to clarify that a chain of events is being described.

OG-79:Defense attorneys have occasionally argued that their clients' misconduct stemmed from a reaction to something ingested, but in attributing criminal or delinquent behavior to some food allergy, the perpetrators are in effect told that they are not responsible for their actions.
A. in attributing criminal or deliquent behavior to some food allergy,
B. if criminal or delinquent behavior is attributed to an allergy to some food,
C. in attributing behavior that is criminal or delinquent to an allergy to some food,
D. if some food allergy is attributed as the cause of criminal or deliquent behavior,
E. in attributing a food allergy as the cause of criminal or delinquent behavior,
--------------------------------
选B。关于79题这里修饰对象的考量(stacey):
so what is it modifying? The phrase is describing an action - someone or something is attributing criminal or delinquent behavior to a food allergy. Who or what is doing that? Logically, the defense attorneys are attributing bad behavior to a food allergy - that matches the info in the first half of the sentence. Hmm. But the second half doesn't mention the defense attorneys; it only mentions the perpetrators. Are the perpetrators attributing their own bad behavior to food allergies? I suppose that's possible, but that's not what the first part of the sentence says. The first part says that the defense attornies are doing this. So I've got a "bad" meaning here - a misplaced modifier.
关于attribute的用法:The correct idiom in the active voice is one attribute x(an effect)to y(a cause)
                                               in the passive voice,x(the effect) is attributed to y(the cause).

Manhattan里面关于 attribute的习语用法归纳:
RIGHT:We ATTRIBUTE the uprising TO popular discontent.
WRONG:We ATTRIBUTE the uprising AS popular discontent.


OG-103:Yellow jackets number among the 900 or so species of the world's social wasps, wasps living in a highly cooperative and organized society where they consist almost entirely of females - the queen and her sterile female workers.
A. wasps living in a highly cooperative and organized society where they consist almost entirely of
B. wasps that live in a highly cooperative and organized society consisting almost entirely of
C. which means they live in a hghly cooperative and organized society, almost all
D. which means that their society is highly cooperative, organized, and it is almost entirely
E. living in a society that is highly cooperative, organized, and it consists of almost all
--------------------------------------------------
选B。其中A选项OG给出的解释:they, reperring to wasps, is an incorrect subject for consist.

2. argue for
    advocate + n.

3. Act + like:means to behave or comport oneself and describes the action of a person:He acted like a fool.
    act + as: describes the function of a thing:the breakwater …act as a buffer. As an inanimate object, it must be performing some function。

manhattan关于 act 的一些知识点:
act as = function as
act like = behave in a similar manner

4. 关于with的用法:
Now let’s talk about the comma + with modifiers. So here we are talking about prepositional phrases. “with” modifiers are very versatile modifiers. They can modify either the preceding clause or the preceding nouns. What they modify actually depends on the context of the sentence and the wording of the modifier itself.
Bihar is India's poorest state, with an annual per capita income of $111.
In this sentence, with modifier actually modifies the preceding noun. With modifier in this sentence has the following sense. Notice how “which has” can be understood to replace “with”.
India’s poorest state, which has an annual per capita income of $111
However, this modifier can be understood to modify the subject of the clause as well because of the nature of the verb - is. This is a linking verb, which establishes the following relationship:
Bihar = India’s poorest state.
右续:

Now lets consider a scenario in which “with” modifier modifies the preceding clause. This is a GMATPrep Question. You can find the detailed solution of this question at this link.
Visitors to the park have often looked up into the leafy canopy and seen monkeys sleeping on the branches, with arms and legs hanging like socks on a clothesline.
The comma +with modifier above modifies the preceding clause. In essence this sentence can be written as two separate sentences:
1. Visitors to the park have often looked up into the leafy canopy and seen monkeys sleeping on the branches.
2. Their arms and legs are hung like socks on a clothesline.
So sentence 2 has been converted into with modifier. This modifier extends the thought of the preceding clause by providing a detail supporting it.
Now if the above sounds very complicated, then do not worry about it. As long as you know that these modifiers are versatile and hence can modify preceding clause and preceding nouns, you would be fine. Let the meaning of the sentence guide you. You should understand the meaning of the sentence and ensure that one of these roles fit well.

5. 在英文中,否定句只有两个标志:否定词no, not.
正确举例:
I have never been to US, Japan and Europe.(肯定句)
I have not been to US, Japan or Europe.(否定句)

6. 注意:任何动词都不可能跟sb.+ that 从句,比如require sb. that ……必错! — 好像真的诶

7. 分词的省略结构
1.分词doing/done(doing 表主动,即动作的发出者;done 表被动,即动作的承受者)
2.介词+分词
3.连词+分词
考点只有一个:分词的逻辑主语必须是后面句子的主语。— 不很确定,需要再观察

8. 代词敏感!! — 需要再观察
把原文中得一个代词换成一个名词,典型的正确答案变换方式。
出现名词的选项,要比出现代词的选项,正确概率高得多!
新名词(原文中没有出现的名词),更是正确选项的标志。
地板
 楼主| 发表于 2013-8-25 07:46:29 | 只看该作者
8月25日(日)
终于找出了独立宣言,一直想仔细看看,贴在这里作偶尔休闲的课外读物

IN CONGRESS, JULY 4, 1776
The unanimous Declaration of the thirteen united States of America
When in the Course of human events it becomes necessary for one people to dissolve the political bands which have connected them with another and to assume among the powers of the earth, the separate and equal station to which the Laws of Nature and of Nature's God entitle them, a decent respect to the opinions of mankind requires that they should declare the causes which impel them to the separation.
We hold these truths to be self-evident, that all men are created equal, that they are endowed by their Creator with certain unalienable Rights, that among these are Life, Liberty and the pursuit of Happiness.  — That to secure these rights, Governments are instituted among Men, deriving their just powers from the consent of the governed,  — That whenever any Form of Government becomes destructive of these ends, it is the Right of the People to alter or to abolish it, and to institute new Government, laying its foundation on such principles and organizing its powers in such form, as to them shall seem most likely to effect their Safety and Happiness. Prudence, indeed, will dictate that Governments long established should not be changed for light and transient causes; and accordingly all experience hath shewn that mankind are more disposed to suffer, while evils are sufferable than to right themselves by abolishing the forms to which they are accustomed. But when a long train of abuses and usurpations, pursuing invariably the same Object evinces a design to reduce them under absolute Despotism, it is their right, it is their duty, to throw off such Government, and to provide new Guards for their future security.  — Such has been the patient sufferance of these Colonies; and such is now the necessity which constrains them to alter their former Systems of Government. The history of the present King of Great Britain is a history of repeated injuries and usurpations, all having in direct object the establishment of an absolute Tyranny over these States. To prove this, let Facts be submitted to a candid world.
He has refused his Assent to Laws, the most wholesome and necessary for the public good.
He has forbidden his Governors to pass Laws of immediate and pressing importance, unless suspended in their operation till his Assent should be obtained; and when so suspended, he has utterly neglected to attend to them.
He has refused to pass other Laws for the accommodation of large districts of people, unless those people would relinquish the right of Representation in the Legislature, a right inestimable to them and formidable to tyrants only.
He has called together legislative bodies at places unusual, uncomfortable, and distant from the depository of their Public Records, for the sole purpose of fatiguing them into compliance with his measures.
He has dissolved Representative Houses repeatedly, for opposing with manly firmness his invasions on the rights of the people.
He has refused for a long time, after such dissolutions, to cause others to be elected, whereby the Legislative Powers, incapable of Annihilation, have returned to the People at large for their exercise; the State remaining in the mean time exposed to all the dangers of invasion from without, and convulsions within.
He has endeavoured to prevent the population of these States; for that purpose obstructing the Laws for Naturalization of Foreigners; refusing to pass others to encourage their migrations hither, and raising the conditions of new Appropriations of Lands.
He has obstructed the Administration of Justice by refusing his Assent to Laws for establishing Judiciary Powers.
He has made Judges dependent on his Will alone for the tenure of their offices, and the amount and payment of their salaries.
He has erected a multitude of New Offices, and sent hither swarms of Officers to harass our people and eat out their substance.
He has kept among us, in times of peace, Standing Armies without the Consent of our legislatures.
He has affected to render the Military independent of and superior to the Civil Power.
He has combined with others to subject us to a jurisdiction foreign to our constitution, and unacknowledged by our laws; giving his Assent to their Acts of pretended Legislation:
For quartering large bodies of armed troops among us:
For protecting them, by a mock Trial from punishment for any Murders which they should commit on the Inhabitants of these States:
For cutting off our Trade with all parts of the world:
For imposing Taxes on us without our Consent:
For depriving us in many cases, of the benefit of Trial by Jury:
For transporting us beyond Seas to be tried for pretended offences:
For abolishing the free System of English Laws in a neighbouring Province, establishing therein an Arbitrary government, and enlarging its Boundaries so as to render it at once an example and fit instrument for introducing the same absolute rule into these Colonies
For taking away our Charters, abolishing our most valuable Laws and altering fundamentally the Forms of our Governments:
For suspending our own Legislatures, and declaring themselves invested with power to legislate for us in all cases whatsoever.
He has abdicated Government here, by declaring us out of his Protection and waging War against us.
He has plundered our seas, ravaged our coasts, burnt our towns, and destroyed the lives of our people.
He is at this time transporting large Armies of foreign Mercenaries to compleat the works of death, desolation, and tyranny, already begun with circumstances of Cruelty & Perfidy scarcely paralleled in the most barbarous ages, and totally unworthy the Head of a civilized nation.
He has constrained our fellow Citizens taken Captive on the high Seas to bear Arms against their Country, to become the executioners of their friends and Brethren, or to fall themselves by their Hands.
He has excited domestic insurrections amongst us, and has endeavoured to bring on the inhabitants of our frontiers, the merciless Indian Savages whose known rule of warfare, is an undistinguished destruction of all ages, sexes and conditions.
In every stage of these Oppressions We have Petitioned for Redress in the most humble terms: Our repeated Petitions have been answered only by repeated injury. A Prince, whose character is thus marked by every act which may define a Tyrant, is unfit to be the ruler of a free people.
Nor have We been wanting in attentions to our British brethren. We have warned them from time to time of attempts by their legislature to extend an unwarrantable jurisdiction over us. We have reminded them of the circumstances of our emigration and settlement here. We have appealed to their native justice and magnanimity, and we have conjured them by the ties of our common kindred to disavow these usurpations, which would inevitably interrupt our connections and correspondence. They too have been deaf to the voice of justice and of consanguinity. We must, therefore, acquiesce in the necessity, which denounces our Separation, and hold them, as we hold the rest of mankind, Enemies in War, in Peace Friends.
We, therefore, the Representatives of the united States of America, in General Congress, Assembled, appealing to the Supreme Judge of the world for the rectitude of our intentions, do, in the Name, and by Authority of the good People of these Colonies, solemnly publish and declare, That these united Colonies are, and of Right ought to be Free and Independent States, that they are Absolved from all Allegiance to the British Crown, and that all political connection between them and the State of Great Britain, is and ought to be totally dissolved; and that as Free and Independent States, they have full Power to levy War, conclude Peace, contract Alliances, establish Commerce, and to do all other Acts and Things which Independent States may of right do.  — And for the support of this Declaration, with a firm reliance on the protection of Divine Providence, we mutually pledge to each other our Lives, our Fortunes, and our sacred Honor. http://www.ushistory.org/declaration/document/

8月27日(一)
OG修饰语部分
Prep2012 35道题,是前面简单到中等难度的题目,错9题(但说实话,中等难度和简单的题目错误率差不多的....)


1. OG
OG-126:The use of lie detectors is based on the assumption that lying produces emotional reactions in an individual that, in turn, create unconscious physiological responses.
A. that, in turn, create uncounscious physiological responses
B. that creates unconscious physiological responses in turn
C. creating, in turn, unconscious physiological responses
D. to creat, in turn, physiological responses that are unconscious
E. who creates unconscious physiological responses in turn
------------------------------------
选A,creat表示修饰复数reactions。对于C选项,OG解释: creating seems to refer back to lying; if used as a participial, creating would have to be preceded by a comma.
从意思上来解读整题(E-GMAT):

Intended meaning from Choice A:
1. Use of lie detectors is based on a certain assumption. Here is the assumption
2. Lying produces emotional reactions in an individual
3. These emotional reactions then create unconscious physiological responses.

Note that “that clause” in this sentence clearly modifies – emotional reactions. Yes, typically relative pronoun modifiers modify the closest noun. However, they can also modify slightly far away noun when this noun is the head of the noun phrase and when such modification makes sense.(从单复数也可以判断出来)
E选项的分析:
Grammatically Correct but Incorrect choice (E) – Notice how in choice E the test makers have changed the modifier from “that clause” to “who clause”. See, “who” modifiers cannot modify things. They can only modify people. So by changing ‘thing’ modifying modifier to ‘person’ modifying modifier, now automatically the entity being modified changes to “individual”. Now the modifier no longer modifies “emotional reactions”. This changes the meaning. This results in different yet still logical meaning.
The intended meaning communicated by Choice E is the following:

1. Use of lie detectors is based on a certain assumption. Here is the assumption:
2. Lying produces emotional reactions in an individual
3. The individual then creates unconscious physiological responses.

Note that without subject matter knowledge, the above cause and effect sequence appears to be logical. Lying produces certain reactions in the individual and then the individual unknowingly creates certain type of responses.
Thus, the meaning communicated by this choice is certainly different from the meaning communicated by choice A. Hence, while choice E is grammatically correct, it changes the logical intended meaning and is not the correct choice.
C选项的错误(stacey):
It's ambiguous here - "individual" is a candidate for the noun, because an individual can create unconscious responses... and so can the reactions... and, hey, so can studying! Which is it? Ambiguity = bad.(右续)
关于这里in turn位置的解释,其实还有一个misplaced modifier的issue:

首先要了解这个in turn在原句是修饰谁的..就是修饰create,如果in turn不好理解可以换成then,其实就是用来indicate动作的先后顺序
看看B选项的in turn放在了名词responses 后面,create那个地方就失去了先后的意思…而responses这里的意思就变为了一系列的responses in turn.

2.Prep12
05:In Alexandria, the Ptolemys founded a library so as the written remains of Greek literature could be gathered together and systematized.
A. so as
B. in which
C. and
D. whereby
E. by which
---------------------------------------------------------------
选B。句子表示Ptolemys的目的是找到一个library as a location where Greek literature could be gathered together and systematized。B选项中,因为library是一个location,所以in which符合原意。D选项中,whereby indicates that the library is a means of gathering and systematizing, instead of a place where this happens.
E选项中,By which indicates that library is an agent of gathering and systematizing, which makes no sense.


10:Though the artifacts of pre-Columbian civilization created a stir from the very first European contacts with the New World in the sixteenth century, it was not until the latter half of the nineteenth century that Western designers, and crafters were inspired to imitate them.
A. it was not until the latter half of the nineteenth century that Western designers, and crafters were inspired to imitate them.
B. they did not inspire imitations of Western designers, artists, and crafters until it was the latter half of the nineteenth century
C. not until the latter half of the nineteenth century was it that they have inspired Western designers, artists, and crafters to imitation
D. until the latter half of the nineteenth century Western designers, artists, and crafters have not been inspired to imitate
E. they were not inspiration of Western designers, artists, and crafters imitations until the latter half of the nineteenth century
-------------------------------------------------------------
选A。The point of this sentence is to assert that although pre-Columbian artifacts were enthusiastically received by Europeans in the sixteenth century, a long period of time elapsed before these artifacts were once again a source of inspiration to Western artisans.
A选项中,the sentence correctly uses past tense for both past, completed time frames - sixteenth and nineteenth centuries.
E选项,the sentence is wordy, and it requires the possessive form of designers, artists, and crafters.


13:In February 1995, Doris Schopper became president of the international council of M~S~F~, known in English as "Doctors Without Borders," the world's largest independent, internatinal, voluntary emergency medical assistance and relief organization.
A. known in English as "Doctors Without Borders,"
B. the English for "Doctors Without Borders,"
C. in English names as "Doctors Without Borders,"
D. "Doctors Without Borders," as they are known in English
E. or "Doctors Without Borders," which is in English
------------------------------------------------------------------
选A。The adjective phrase known in English..correctly describes the international council named in French.
C选项中,The phrase named as...is a departure from the idiomatic named....。E选项中the placement of the modifier in English makes it an adjective describing the world's...organization instead of the phrase Doctors without Borders.


14:Restorers say that if allowed to remove and replace the discolored layer of varnish on the Mona Lisa, the colors Leonardo da Vinci painted nearly five hundred years ago will once again shine through.
A. if
B. if it is
C. if they are
D. when
E. when it is
--------------------------------------------------------------------
选C。

3. architects and stonemasons是指人,不是东西
architect建筑师
architecture建筑学,建筑风格

5#
 楼主| 发表于 2013-8-28 13:37:48 | 只看该作者
8月28日(三)
开始背GRE单词
新东方语法笔记第二遍
Prep2012第36-75题,40题中至难级别的题竟然只错5题,激动五分钟...会好好总结错题及拿不准的题
数学JJ 1-100

1. 名词后跟that 还是of? :n.+that (解释) n.+of (限制);例如Hypothese
如果后面的划线部分是在解释这个名词,那就跟that
如果后面的划线部分是在限制这个名词,那就跟of.

2. allow1、allow sb. to do sth.(一般正确选项都是这个)
2、allow that 这时不是允许的意思,而是“承认、认可”

3. 极度抽象化名词只能跟of.比如:importance/necessity/need,因为这些抽象名词,不可能或无法被解释.

4. more+adj.+n. 会产生歧义,因为more 既可以理解为修饰adj.,也可以理解为修饰n

5. 助动词、介词尽量补出原则

8月29日(四)
阅读JJ 21-30

8月30日(五)
新东方语法笔记第三遍
GRE单词背了1/10...不容易啊,六千多个啊...


1. 英语发展过程
首先是实义名词——动词——形容词——抽象名词——分词或动名词——从句
抽象名词起源于:动词;或者 形容词
简介语言普遍规律:
动词>形容词>抽象名词>分词或动名词>从句
conversion(抽象名词)优于being converted(分词结构)

2.GMAT 中有5 个单词是"涨"的意思:
soaring; rise ;raise; increase; grow
注意语义重复

3. 食物过敏
1.总称: food allergies
2.对某种食物过敏an allergy to some food

4. 介词不是考试重点。

5.
regard as
begin as
perceive as
think of as
be prized as
be acclaimed as (被称作)
see as(看做)
represent as
depict as(描述)

6. order
(1)order sb. to do sth
(2)order that 虚拟语气(should 必须被省略)
(3)order sth.(点菜,点某物)

7. be able to be done 必错!,因为无法“有能力被……”

9月1日(日)
GRE单词已经背过1/6
数学JJ 101-200
看自己的备考日记



6#
 楼主| 发表于 2013-9-3 01:09:53 | 只看该作者
9月2日(一)
GRE单词背了1/5
数学JJ粗略过完一

总结Prep2012
新东方语法笔记放在下面,可以经常过,批判着看


1. Prep 2012
Prep2012-20:Unlike computer skills or other technical skills, there is a disinclination on the part of many people to recognize the degree to which their analytical skills are weak.
A. Unlike computer skills or other technical skills, there is a disinclination on the part of many people to recognize the degree to which their analytical skills are weak.
B. Unlike computer skills or other technical skills, which they admit they lack, many people are disinclined to recognize that their analytical skills are weak.
C. Unlike computer skills or other technical skills, analytical skills bring out a disinclination in many people to recognize that they are weak to a degree.
D. Many people, willing to admit that they lack computer skills or other technical skills, are disinclined to recognize that their analytical skills are weak.
E. Many people have a disinclination to recognize the weakness of their analytical skills while willing to admit their lack of computer skills or other technical skills.
---------------------------------------------
选D。D选项making people the subject of the sentence allows a construction that clearly contrasts how they feel about the two sets of skills。C选项中,the construction skills bring out a disinclination in many people is wordy, awkward, and idiomatically incorrect.
关于bring out: It's warm enough to bring out the garden chairs.
She was so shocked that she could hardly bring out a word.
I want to bring out clearly all the issues that are involved.


30:A recent poll of elected officials suggests that candidates, when in the midst of a tough campaign, often make statements about an opponent that they may not think is ture.
A. is
B. are
C. to be
D. of as
E. it is
--------------------------------------------------------
选B。主语是谁?是statements....

33. The infant mortality rate has decreased steadily over the past decades and is now at a lower rate than ever before.
A. now at a lower rate than
B. now lower than
C. now a lower rate than
D. presently lower than it was
E. presently lower than the rate was
-------------------------------------------------------
选B。还是看主谓啊,the infant mortality rate has...and is....所以不能再跟at a lower rate了

44. In 1982, archaeologists recovered Henry VIII's ship Mag Rose, which, having sunk in 1545, with more than 17,000 artifacts on board, everything from arrows and longbows to a barber-surgeon's chest containing items commonly used by sixteen-century medics.
A. In 1982, archaeologists recovered Henry VIII's ship Mag Rose, which, having sunk in 1545, with more than 17,000 artifacts on board,
B. Henry ship Mary Rose sank in 1545 and, recovered by archaeologists in 1982, they found more than 17,000 artifacts on board,
C. When Henry ship Mary Rose, which sank in 1545, was recovered in 1982, archaeologists found more than 17,000 artifacts on board,
D. Having sunk in 1545, Henry VIII's ship Mary Rose was recovered in 1982 by archaeologists, finding more than 17,000 artifacts on board, with
E. Finding more than 17,000 artifacts on board, Henry ship Mary Rose, which sank in 1545, was recovered in 1982 by archaeologists and it had
-----------------------------------------------------------
选C。小心得,看B选项,莫名其妙出来they的,就要想到底指代谁,D选项中一看Having就想难道一直在沉吗,沉到现在?
A选项,the clause starting with which is ungrammatical because it needs a main verb: sank rather than having sunk。C选项正确,the introductory dependent clause provides a subject Mary Rose and a main (infinitive) verb was recovered. The independent clause also provides a subject archaeologists and main (infinitive) verb: found


45. In undisturbed primary forests, the Honduran mahogany grows up to a height of 130 feet, having a buttressed trunk and a crown that spread over the canopy of lesser trees.
A. up to a height of 130 feet, having a buttressed trunk and a crown that spread
B. up to 130 feet in height, and with a buttressed trunk and a crown spreading
C. to as high as 130 feet in height, having buttressed trunk and with a crown that spread
D. to a height of 130 feet, with a buttressed trunk and with a crown that spreads
E. as high a height as 130 feet, having a buttressed trunk and a crown spreading
------------------------------------------------------------------
各种重复,所以记下来这道题。选D。
The sentence describes the Honduran mahogany tree in terms of height, trunk structure, and foliage. In rewritten versions, these descriptors are presented in excessively wordy and repetitious phrases. Some of the versions are worded in such a way that spread could refer illogically to both the trunk and the crown.
A. the word up is redundant, because trees can only grow in one direction, up to a height. The relative pronoun that refers to the sigular noun crown, and therefore requires a singular verb spreads.
B. the conjunction and anticipates a new clause but instead is followed by a prepositional phrase, leaving the second clause without a verb.
C. the words as high as are redundant because they say the same thing as in height. The preposition with is unnecessary and nonsensical in the participial phrase having...
D. correct. this version of the sentence doesn't suffer from the wordiness of the others. The second with also makes it clear that spread refers only to the crown and not to the buttressed trunk.
E. the phrase as high a height as is wordy and redundant.


2. 曼哈顿关于Enough的用法
RIGHT: The book was SHORT ENOUGH TO READ in a night.
            The book was SHORT ENOUGH FOR me TO READ in a night.
SUSPECT: The power plant has found a way to generate energy at an unprecedented scale, ENOUGH FOR powering and entiry city.
WRONG: The book was SHORT ENOUGH THAT I could read it in a night.
              The book was SHORT ENOUGH FOR IT TO BE read in a night.
              The book was SHORT ENOUGH SO THAT I could read it in a night.
              The book was SHORT ENOUGH AS TO BE read in a night.

3. 曼哈顿关于Doubt的用法
RIGHT: We DO NOT DOUBT THAT the apples are ripe.
           We HAVE DOUBT THAT the apples are ripe.
            She DOUBTS WHETHER Jan will arrive on time.
SUSPECT: She DOUBTS THAT Jan will arrive on time.
WRONG: We DO NOT DOUBT WHETHER the apples are ripe.
             We HAVE NO DOUBT WHETHER the apples are ripe.

4. have seen,不是have saw...

9月4日(三)
数学JJ
错题总结
一点点高智
威逻辑总结:
逻辑题目概述及总原则:
1.请关注推理过程中的GAP  (话题相关性)
2.请关注推理过程中的主体对象
3.格外关注推理过程中含有的比较 (相对量与绝对量)

逻辑基本概念:
1. Argument=premise论据+conclusion (P支持C)
2. 逻辑文章只有一个conclusion,其他全是支持C的P

削弱题:
基本概念:
1. Argument=premise论据+conclusion P支持C
2. 逻辑文章只有一个conclusion,其他全是支持CP
做题方法:
1.找到文章的PC
1.1利用因果关系寻找,如果没有,利用句间支持关系,例如冒号后面一定是P
1.2 P是相对客观的,而C是相对主观的,例如实验调查研究的结果是事实P,而含有情态动词例如should更有可能是C
1.3 读完文章后能够简单概括的写出PC
1.4 复杂文章在回看时,重点回看C和重要的主P
2.思考:正确答案必须给一个理由反对结论,话题与P越贴近越优
2.1 遇到复杂的结论,例如plan; goal; if a,b; by a,b找到需要反对的点,一般反对goalb的地方,其他信息全为事实不能反对
*3.如果文章是原因型结论,正确选项反对PC之间的关系,可能提出他因或者举反例
4.排除选项时请格外关注:选项中含有的比较,别的人群、别的事件、别的地点,模糊的表达如sometimes/not all/possible
假设题Assumption:一种特殊的P
Hypothesis更倾向于C
基本概念:
1.问题标志词: assume, assumption,presuppose, additional premise, not true unless, depend on, rely on
2.Assumption是文章结论成立的必要条件(没它就不行)
3.假设有两个特征,一是弥补GAP*二是not weaken(重点考察)
做题方法:
1.找到文章的PC并理解
2.优先考虑否定句的选项
3.把这些选项取反,削弱文章的就是正确答案(推翻C或者推翻PC关系)
4.P越相关越好,要格外注意选项中含有的比较,别的事物别的人群别的地点,含有极端词的选项不好,越模糊越好
评价题:
基本概念:1.对于选项中问题的不同回答会对文章一方面产生削弱另一方面产生支持的作用
         2.评价是削弱和假设的结合体
做题方法:1.找到文章的PC并理解
         2.判断是否为原因型论证
         3.回答选项中的问题,看其是否有可能削弱文章(削弱C或者削弱PC关系)
         4.注意事项和削弱题一样(模糊和极端的表达均不是太好)
解释题:paradox
基本概念:1.解释题的文章不是PC,而是一个paradox
         2.解释题的文章是A howeverB,正确选项把矛盾的两件事变得不矛盾
做题方法:1.找到文章的矛盾A however B
            其实就是一个写好的P however CX
         2.AB中间加一个条件C,使得AB不矛盾
           2.1解释题的选项和削弱题一样
      附加(初学者不用):如果是原因型论证改过来的P however CX,解释的重点是P
           2.2如果AB是对比关系,加入的条件C要体现AB的不同之处
读文章前应该:根据关键词找到P和C所在的语句
读文章时应该:简洁简单输入最重要的信息不要太在意细节
读文章后应该:思考正确选项的“目的”

填空题:
基本概念:1.填空题绝大部分填的是文章的P
做题方法:P therefore C,since_______  弥补GAP,类似假设题(取反之后会削弱)
          P however CX, since_______  类似解释题,弥补PCX之间的GAP
1.Best不代表好 not best不代表不好 注意not+极端词
2.only if A,BBA,通常理解成非A推非B
3.要注意选项中含有的时间
4.看到百分比要思考基数 看到价格想到供求关系
5.当文章在说实验调查研究时,如果样本明显,应考虑样本问题

如何判断原因型结论:
1. C中在探讨一件事情发生的原因(C中有attribute, result from, cause,this is the reason responsible
2. 如果文章是明显的实验、调查、研究、统计,就很有可能是
3. 如果结论含有should    plan goal   if A, B一般不是
句子作用题:
基本概念:1.句子的作用包含句子的态度以及句子的角色
          2.
做题方法:1.分析句子的态度:
   找到整句间的转折关系例如:A-与作者态度不同) however B+与作者态度相同)
   A+ however B- however C+
   but yet although: 句间 不考虑 但大写的要考虑
   2.判断句子的角色(是P还是C)通过关键词 因果关系 主观客观 情态动词等判断
premise evidence reason exampleconsideration explanation context assumption
conclusion position claim judgmentprediction hypothesis
   *有时需要读句子意思来判断句间关系
   *若文中没有句间转折,选项不能出现反对的词汇
   *句首的负面评价也是句间转折

归纳题
基本概念:
1.问题标志词:conclude/conclusion   infer(合情合理即可)   must be true(推理很完美)   support(被文章支持,要求较低)
2.归纳题的绝大部分文章没有C只有P
3.正确选项的信息应该能回到原文找到依据
4.正确答案一定要和主P相关,跟其他越多P相关越好
做题方法:
5.找到文章的主P
6.排除与主P无关的,与文中信息相反的,文中找不到依据的
7.判断剩余选项,哪个是主P或者是主P结合其他P的推论
3.1 选项中假定好的事实例如if,也是文中重要的P
3.2 如果文章有结论,把结论看成主P往前推导即可
3.3 must be true要求完美的推理,infer support可以不完美,support有时只需要态度一致,内容相似即可
加强题
基本概念:
1.假设属于加强当中的一种,非常类似(取反之后也能削弱)
2.加强题的本质是在PC之间弥补新P,新P不一定是必要条件或者完美弥补GAP,是他们的“降级”表达
做题方法:
1.找到文章的PC并理解
2.判断是否为原因型论证
2.1 如果是,通常用否命题例子、类比例子来加强,如果不是,参见第3
3.如果PCGAP明显,直接弥补,正确答案与之类似;如果GAP不明显,思考not weaken
注意:1.P越相关越好
     2.取反之后能削弱是判断加强的唯一标准
总结
削弱:1.找到PC
      1.1 回看时,只读重点
      1.2 实验调查结果算P
      1.3 简洁输入
      1.4 读完文章后,记住P和CX
     2.判断原因型结论
      2.1 C中说原因
      2.2 实验调查科学家responsible attribute等词汇
      2.3 含有should    plan goal  if A,B    by A,B的结论就不是
   是:削弱PC关系,找他因或者举反例不是:削弱C但是承认P---与P越相关越好
评价  1.找到PC 2.判断原因型结论 3.对选项中的问句进行回答,其中一种回答可以削弱
假设  1.找到PC  2.判断原因型结论 3.优先考虑否定句选项,取反之后可削弱
加强  1.找到PC 2.判断原因型结论 3.取反之后可削弱
填空  p therefore c, since_______ (加强)
     P however cx, since________ (解释)
解释  1.找到文中的P however CX  2.*    3.在P和CX之间放一条件让他们不矛盾
归纳  1.找到主P  2.排除  3.判断是否是主P和其他P的推论

需要格外注意的选项:
1.含有比较的选项,注意谁跟谁比,他们之间的关系是否与文章相关
2.别的事物别的人群别的地点---->不好
3.模糊、极端词的选项:削弱的模糊词不好,假设的极端词不好
注意事项:
1.看到百分比要思考基数问题,以及与数量不一样
2.看到实验明显的样本,要思考样本差异
3.Not+极端词,not+比较级,no more, A only if B
4.问题中含有support, provide ground有可能是归纳、加强、解释
5.多背单词

本帖子中包含更多资源

您需要 登录 才可以下载或查看,没有帐号?立即注册

x
7#
 楼主| 发表于 2013-9-4 15:42:02 | 只看该作者
续9月4日(三)
8#
发表于 2015-10-12 15:18:57 | 只看该作者
楼主有心人啊!
9#
发表于 2016-5-22 12:40:02 | 只看该作者
楼主好仔细
10#
发表于 2016-5-23 15:04:33 | 只看该作者
顶楼主!               
您需要登录后才可以回帖 登录 | 立即注册

Mark一下! 看一下! 顶楼主! 感谢分享! 快速回复:

手机版|ChaseDream|GMT+8, 2024-4-28 02:21
京公网安备11010202008513号 京ICP证101109号 京ICP备12012021号

ChaseDream 论坛

© 2003-2023 ChaseDream.com. All Rights Reserved.

返回顶部